Download as pdf or txt
Download as pdf or txt
You are on page 1of 42

RADIANT NOTES RADIANT NOTES

FCPS PEARLS
11th Edition-2022
“Don't sit down and wait for the opportunities to come.
11 Edition-2022
Get up and make them.” – Madam C.J Walker

“All dreams are within reach. All you have to do is keep


moving towards them.” – Viola Davis

“Don’t be pushed around by the fears in your mind. Be


led by the dreams in your heart.” – Roy T. Bennett

DR. RAFI ULLAH | DR. MAHREEN MALIK | DR. AMNAH ZUBAIR| DR. MARIA KHAN
|DR. HAMID ATHAR| DR. ALBER SOOMRO|

0|Page
RADIANT NOTES FCPS PEARLS 11th Edition-2022

PUBLISHER:

SWAT BOOK HOUSE & PUBLICATIONS


For Home Delivery All Over Pakistan
Whatsapp# 0346-9049087, Phone# 0946-726817
We deal in ALL KINDS of Medical Books, Medical Instruments & Lab Coats etc.

SWAT BOOK HOUSE Offers FREE HOME DELIVERY ALL OVER PAKISTAN
(TCS CASH ON DELIVERY)
Pre-Booking will start from Tuesday 18th January InshaAllah
For Pre-Booking of Radiant Notes FCPS PEARLS 11th Edition 2022 by Dr Rafiullah

Please Fill This Proforma and Whatsapp to 0346-9049087

Book Name: FCPS Pearls 11th Edition 2022


Number of Books needed=
Name:
Complete Address:
City:
Cell#

1|Page
RADIANT NOTES FCPS PEARLS 11th Edition-2022

SALIENT FEATURES OF Radiant Notes FCPS Pearls 11th Edition (2022)


 The Famous Golden Points completely updated and Arranged
CHAPTERWISE for the very first time

 Contains All Golden Files from Zero to 13


i.e. Golden File Zero,1, 2, 3,4,5,6,7,8,9,10,11,12 & 13
 Contains All the Past Papers from 2015 till December 2021

 Golden File 12 & 13 Contains Latest Papers of February, June,


August and November 2021

 CPSP Demo Questions


 Covers All Specialities
 100% Errata Free & Coloured

2|Page
RADIANT NOTES FCPS PEARLS 11th Edition-2022

Table of Contents
Section: 1---CPSP DEMO Questions Page#
CPSP Demo Questions with CPSP keys 9

Section: 2---Golden File Special –contains Golden file 1-11


Calculations 21
Cell Physiology 25
Gastroenterology 32
Pulmonology 46
Cardiovascular 64
Nephrology 87
Hematology 99
Oncology 111
Endocrinology 120
Neurology (Neuroanatomy and Neurophysiology) 135
Musculoskeletal System 170
General Pathology 176
Immunology 193
Microbiology 199
Histology 212
Biochemistry 217
Pharmacology 221
Embryology 231
Upper Limb 241
Lower Limb 248
Thorax 256
Abdomen, Pelvis and perineum 260
Spinal Cord and General 271
Public Health Sciences and General 275
Remaining Questions of Golden file 10 and 11 285

Section: 3---Golden File 12---Feb and June 2021 Papers


Medicine & Allied 16th Feb 2021 Afternoon 304
Medicine & Allied 16th Feb 2021 Night 321
Medicine & Allied 17th Feb Night 339
Medicine & Allied 14th June, Night 356
Medicine & Allied 16th June Afternoon 373
Medicine & Allied 16th June Night 388
Medicine & Allied 17th June Morning 405
Medicine & Allied 17th June Afternoon 421
Medicine & Allied 17th June Night 437
Surgery & Allied 17th Feb Morning 453
Surgery & Allied 17th Feb Afternoon 464
Surgery & Allied 17th Feb Night 479
Surgery & Allied 15th June Afternoon 492
Surgery & Allied 15th June Night 505
Surgery & Allied 16th June Morning 520
Surgery & Allied 16th June Afternoon 535
Gynae and Obs 17th Feb Morning 547
3|Page
RADIANT NOTES FCPS PEARLS 11th Edition-2022

Gynae and Obs 17th Feb Afternoon 561


Gynae and Obs 15th June Morning 573
Gynae and Obs 15th June Afternoon 589
Radiology 16th Feb Afternoon 605
Radiology 14TH June Afternoon 617
Radiology 14TH June Night 630

Section: 4---Golden File 13---Sep and Nov-Dec 2021 Papers


Medicine & Allied 29th Aug, Afternoon Shift 643
Medicine & Allied 30th Aug, Afternoon Shift 660
Medicine & Allied 30th Aug, Night Shift 672
Medicine & Allied 31st Aug, Morning Shift 688
Medicine & Allied 31st Aug, Afternoon Shift 704
Medicine & Allied 31st Aug, Night Shift 718
Medicine & Allied 1st Dec, Afternoon Shift 733
Medicine & Allied 1st Dec, Night Shift 749
Medicine & Allied 2nd Dec, Morning Shift 761
Medicine & Allied 2nd Dec, Afternoon Shift 776
Surgery & Allied 30th Aug, Morning Shift 791
Surgery & Allied 30th Aug, Afternoon Shift 805
Surgery & Allied 1st Sep, Morning Shift 822
Surgery & Allied 30th Nov, Afternoon Shift 837
Surgery & Allied 30th Nov, Night Shift 852
Surgery & Allied 1st Dec, Morning Shift 866
Gynae and Obs 29th Aug, Morning Shift 882
Gynae and Obs 30th Aug, Morning Shift 896
Gynae and Obs 1st Dec, Morning Shift 911
Gynae and Obs 1st Dec, Afternoon Shift 926
Radiology 28th Aug, Night Shift 941
Radiology 29th Aug, Afternoon Shift 955
Radiology 30th Nov, Morning Shift 968
Radiology 30th Nov, Afternoon Shift 982

How To Finish This Book In 30 Days


Day 1st CPSP DEMO Questions
Day 2nd -Day 12th 25 points daily from Golden file special plus 2 papers from Golden file 12
Day 13th – Day 24th 25 points daily from Golden file special plus 2 papers from Golden file 13
Day 25th -Day 30th Revision—90 to 100 points daily plus highlighted MCQs from golden file 12 and 13
 While doing papers highlight only those questions which you think would need revision
 Don’t waste your time searching for errata or neither make one for the book---if there is any need
for errata, I would make one

4|Page
RADIANT NOTES FCPS PEARLS 11th Edition-2022

Preface
 I am thankful to Almighty Allah who gave me the courage to write this book. This book is
the most updated book available now for FCPS-1.
 This book has been written to meet the needs and requirements of students appearing in
FCPS Part-1 and as well as other medical license exams. Students have always felt a need for
a book which include all past papers BCQS as well as authentic key of it. Here I have tried
my level best to make such a book and make it error free.
 This book is the only book which contains all the golden files from start till date and I have
always tried to keep it in a single volume; so as to make it easy for students
 This book will help the students in many ways including
Contains CPSP demo questions with CPSP key
Contains All the Past Papers from 2015 till 2020
Contains All the Golden Files Golden File Zero, 1, 2, 3, 4, 5, 6, 7. 8, 9 10 & 11
Golden File 12 & 13 Contains the Recent 2021 Papers, All with Correct Key and Solved
I have tried my best to make this book error free. If any error found please inbox me on our
Facebook page by the name of “Radiant notes & lectures”

This book has been made solely for the purpose of helping the students.
Any suggestions/ corrections/ Or Help in Papers compilation would be highly appreciated
and his/her name will appear in the future edition.
May ALLAH succeed you all.
Remember us in your prayer
Dr. Rafi Ullah
Dr. Mahreen Malik
Dr. Amnah Zubair
Dr. Maria Khan
Dr. Hamid Athar
Dr. Alber Soomro

Helping Hands:
Dr. Hina Gul
Dr. Habiba Sajjad
Dr. Sania Koondhar
Dr. Nadia Talpur
Dr. Seema Hussain Panhwar

5|Page
RADIANT NOTES FCPS PEARLS 11th Edition-2022

Section- 1
CPSP Demo
Questions with CPSP
Key
(Do not mess with the key)

The best preparation for tomorrow is doing your best today

“H. Jackson Brown”

6|Page
1. On H & E staining a student sees the (b) : Iron deficiency anemia
hallow structure around the nucleus. What (c) : Pernicious anemia
it could be: (d) : Thalassemia
(a) (a)Golgi apparatus (e) : Autoimmune hemolytic anemia
(b) Lysosomes Ans: C
(c) Ribosomes 8. A young child has increased BP in upper
(d) Endoplasmic reticulum limbs while decreased BP in lower limbs.
(e) Mitochondria Pulse is also week. What is the probable
Ans: B diagnosis?
(a) : Preductal Coarctation of aorta
2. A patient presents with recurrent epistaxis. (b) : V.S.D
Investigation reveal decreased factors II, (c) : Patent ductus arteriosus
VII, IX, X& protein-C which of the following (d) : Post ductal Coarctation of aorta
is most likely diagnosis of this condition? (e) : Transposition of great vessels
(a) Christmas disease Ans: D
(b) Hemophilia
9. In case of typhoid fever of 06 days. Which
(c) Vitamin-K deficiency
investigation is of first choice?
(d) Von Willebrand disease
(a) : Widal test
(e) Liver dysfunction
(b) : Blood culture
Ans: C
(c) : Typhi-dot test
3. Blood brain barrier is formed by: (d) : Bone marrow culture
(a) Capillary endothelium (e) : Urine culture
(b) Continuous basement membrane Ans: B
(c) Pericytes
10. In a patient with diabetes, which is the best
(d) All of above
antihypertensive drug?
(e) None of above
(a) : Captopril
Ans: A (b) : Diuretic
4. Which of the following fungi produce life (c) : Verapamil
threatening infection in patients with (d) : Propranolol
diabetic ketoacidosis? (e) : Diltiazem
(a) : Candida Albicans Ans: A
(b) : Histoplasmosis 11. Urine Examination of a patient with
(c) : Mucor diabetes demonstrates. Ketone-bodies.
(d) : Aspergillus What is the mechanism of formation of
(e) : Blastomycosis these ketone-bodies?
Ans: C (a) : Insulin deficiency
5. A patient has non-reactive HBsAg, non- (b) : Hyperglycemia
reactive HBeAg & reactive anti HBc Ab. (c) : Defective fat metabolism
This stage of patient indication what? (d) : Hypoalbuminemia
(a) : Acute infection (e) : Defective glucose metabolism
(b) : Carrier Ans: A
(c) : Transient resolving stage of hepatitis 12. After giving blood transfusion, a patient
(d) : Immunized develops hypersensitivity reaction. Which
(e) : Chronic active disease type of hypersensitivity reaction is this?
Ans: C (a) : Type I Hypersensitivity
(b) : Type II Hypersensitivity
6. A Patient has enlarged parotid gland with
(c) : Type III Hypersensitivity
pain in this region. Which nerve is carrying
(d) : Type IV Hypersensitivity
pain fibers from the parotid gland? (e) : A.D.C.C
(a) : Auriculotemporal nerve Ans: B
(b) : Glossopharyngeal nerve
13. A Patient develops lesion in the Caudate
(c) : Facial Nerve
nucleus. Which is most common clinical
(d) : Temporal nerve feature Indicating this lesion?
(e) : Greater auricular nerve (a) : Chorea
Ans: A (b) : Intentional Tremor
7. A Patient has anemia, hyper-segmented (c) : Resting Tremor
Neutrophils on peripheral blood (d) : Hemiplegia
examination & neurological manifestations. (e) : Nystagmus
Which type of anemia he is suffering from? Ans: A
(a) : Folic acid deficiency anemia

7|Page
Section- 2
Golden file Special
Includes Summarized

Golden File Zero (2015 March, April, May


Papers)
Golden File-1 (2015 Sep, Oct, Nov Papers)
Golden File-2 (2016 March, April, May Papers)
Golden file-3 (2016 Sep, Oct, Nov Papers)
Golden file-4 (2017 March, April, May Papers)
Golden file-5 (2017 Sep, Oct, Nov Papers)
Golden file-6 (2018 march & may Papers)
Golden file-7 (2018 Sep &, Nov Papers)
Golden file-8 (2019 February & April papers)
Golden File-9 (2019 September and November)
Golden File-10 (2020 February and July papers)
Golden File-11(2020 Sep and November papers)

Note: Do Read the important note on the next Page


before starting

It always seems impossible until it’s done—


“Nelson Mandela”
8|Page
Important Note
 This file came into idea, after getting a message from students asking for errata, so I
thought I would give them all the golden files with proper text, so they could read and
solve MCQ’S and make their own errata if needed, rather than asking someone else
and wasting their time

 No book could be 100% error free, while recalling MCQ, so here is the best effort
with proper references and text

 The other question that students repeatedly ask is “due to shortage of time, I am
unable to do this golden file or that”, all of the golden files are almost the same with
addition of few new questions every year, so I compiled them all in few pages. (u can
thank me later after passing: p)

 As we all know that every CPSP paper is 80 – 90% repeat in every exam, so why put
an extra effort on spending and repeating the same questions and sometimes with
different answers too, why not let’s put them all together along with theory for proper
understanding.

 As there is no repetition of MCQ’ (InshAllah), so must read this file with proper
understanding at least 2 times before appearing in your exam
 I have tried more than my level best; I hope it proves useful for all of you
 Save your time, save your money---and thank me later
 Any suggestion in this file would be highly appreciated
Good luck

9|Page
Includes August and Nov-Dec 2021 Papers

You’ve got to get up every morning with determination if you’re going


to go to bed with satisfaction.

“GEORGE LORIMER”

10 | P a g e
RADIANT NOTES FCPS PEARLS 11th Edition-2022

Medicine & Allied 29th Aug 2021, c. Flaccid paralysis


d. Dry skin
Afternoon shift e. Babinski sign
1. The risk of Lichen planus changing in to Ans: B
malignancy is?
a. 1-10% 7. A 40 years old man has pain felt deep in the
b. 10-15% nose and face between the mouth and orbit to
c. 11-14% the ear and temporal region. The pain may be
d. 0-5% due to the lesion of the?
Ans: B a. Maxillary branch of the trigeminal nerve
b. Auricular nerve
2. A 65-year male develops right sided hemiplegia c. Facial nerve
and slight deviation of angle of mouth to the d. Hypopharyngeal nerve
left while talking. He is able to close his eyes e. Ophthalmic branch of the trigeminal
on command. What other finding can be nerve
present? Ans: A
a. Wasting of muscle and fasciculation
b. Withdrawal 8. In stage 4 Ca cervix, which group of lymph
c. Lead pipe rigidity nodes are involved?
d. Excessive drooling a. Internal iliac
e. Clasp knife rigidity b. Para aortic
Ans: E c. Deep inguinal
d. Superficial inguinal
3. After head injury in a car accident, pt e. External iliac
developed depression and lack of motivation. Ans: A
This is most likely due to lesion in?
a. Occipital lobe Both internal & external iliac (if only one to choose –
b. Temporal lobe click internal iliac
c. Globus pallidus
d. Parietal lobe 9. A patient present with acute painful swelling in
e. Frontal lobe front of left ear. O/E left parotid gland was
Ans: E enlarged This is most likely due to?
a. Compression of maxillary nerve
4. A 26-year boy presented with pallor and b. Involvement of external acoustic meatus
weakness. O/E he has fever 101'C, gum c. Compression of Auriculotemporal nerve
bleeding, corneal petechia and epistaxis. The
d. Pressure on temporomandibular joint
best test to confirm diagnoses is?
a. CBC e. Fibrous capsule
b. Coagulation profile Ans: C
c. Molecular biology
10. Bacillus anthrax will grow on which culture
d. Bone marrow aspiration
medium?
e. Electrophoresis
a. Loffler
Ans: D
b. Sudan black
5. Colleagues complain about a 30-year woman Ans: B
who forges signs on important documents
11. The character which distinguishes anthrax
and search for belongings of others in their
bacillus from other organisms is:
absence. She has joined another company
and also have started smoking. What test a. Aerobic / Anaerobic growth atmosphere
would you do for her psychiatric evaluation? b. Capsule formation
a. Urine analysis for drugs c. Gram staining
b. B- HCG d. Motility
c. Thyroid e. Position of spore
d. Cortisol Ans: E
Ans: A
12. A 60-year male weighing 70 kg has total body
6. A young farmer while spraying suddenly water of 42L (60% of body weight). How much
collapsed in his fields and was brought to ER. of this will be present in interstial fluid?
O/E he was frothing but no salivation and no a. 42L
excessive lacrimation. He is suspected case b. 3L
of OP poisoning. What other sign will be c. 11.5L
present? d. 28L
a. Bradycardia e. 12L
Ans: C
b. B/L Pin point pupils

11 | P a g e
RADIANT NOTES FCPS PEARLS 11th Edition-2022

 Total body water is 42L a. Inward Ca current


 so, it means ICF (40%) is 28L b. Na and K
 so ECF IS 14L c. Ca and Na
 Now ECF is composed of 3.5L of plasma d. Ca and K
volume and remainder is interstial fluid e. Inward Na current into cells
13. The most characteristic ECG finding showing Ans: E
acute MI is?
a. Displacement of ST segment in the 19. Aspirin toxicity should be treated with?
leads overlying the area of infarct a. Bicarbonate
b. NH4Cl
b. Deep and wide Q wave
c. Diuretics
c. Peaked T waves
d. IV hydration
d. Prolonged PR interval Ans: A
e. Prominent T wave inversion Toxicity Treatment
Ans: A Acetaminophen N-acetylcysteine
(replenishes glutathione)
14. After an injury at upper medial side of arm,
Organophosphates Atropine > pralidoxime
there is loss of flexion at elbow joint and loss
(AChE inhibitors)
of sensation at lateral side of forearm. The
Benzodiazepines Flumazenil
nerve likely injured is?
Heparin Protamine sulfate
a. Axillary
b. Ulnar Iron Deferoxamine, Deferasirox,
c. Musculocutaneous Deferiprone (3D’S)
d. Radial OpiOids NalOxOne
Ans: C Warfarin Vitamin K (delayed effect),
fresh frozen plasma
15. Which of the following is raised (early on) in (immediate)
acute MI in the first hour? Anticholinergic Physostigmine
a. Myoglobin (atropine) toxicity
b. LDH Salicylates NaHCO3 (alkalinize urine),
c. Trop T dialysis
d. CK MB TCAs NaHCO3 (stabilizes cardiac
Ans: A cell membrane
20. A middle-aged woman presented with pain in
16. The substance used to measure total body epigastrium. The physician diagnosed it as a
water is? case of peptic ulcer. The nerve most likely to
a. Thiosulfate be mediating this referred visceral pain is?
b. Inulin a. 9th spinal
c. Evan blue dye b. 10th spinal
d. Antipyrene c. Greater splanchnic
e. Mannitol d. Least splanchnic
Ans: D
e. Lesser splanchnic
Compartment Marker Used to Measure
Ans: C
Volume
TBW Tritiated H2O 21. A patient taking Dicumarol can expect to have
D2O which of the following symptoms?
Antipyrene a. Delayed blood clotting
ECF Sulfate b. Enhanced blood clotting
Inulin
c. Normal blood clotting
Mannitol
d. Plasmin deficiency
Plasma (1/4 of RISA (radioiodinated serum
e. Platelet’s aggregation
ECF) albumin)
Ans: A
Evans blue
17. A lady had colostomy and was put on total 22. The correct composition of blood is?
parenteral nutrition (TPN). She is prone to a. 45% cell, 55% plasma
develop which of the following initially?
b. 70-75% plasma
a. Ketoacidosis
b. Hypocalcemia c. 80% plasma, 20% cells
c. Hypophosphatemia d. 90% cells, 10% plasma
d. Fatty liver Ans: A
e. Hyperglycemia
Ans: E 23. The umbilical artery carries?
a. Oxygenated blood from the placenta
18. Phase 4 depolarization produced in SA node b. Oxygenated blood to the placenta
is due to? c. Deoxygenated blood to placenta

12 | P a g e
RADIANT NOTES FCPS PEARLS 11th Edition-2022

d. Deoxygenated blood from the placenta b. Circumflex


Ans: C c. Left main
 The right and left umbilical arteries, carry d. RCA
deoxygenated blood from the fetus to the e. Obtuse marginal
placenta. Ans: D
 The left umbilical vein, which carries
oxygenated blood from the placenta to the 31. A patient known case of HIV is on
fetus and has highest oxygen saturation immunosuppressive drugs presents with
24. What is definitive evidence for ovulation? high grade fever, neck rigidity, +ve Kerning.
a. Raised body temperature CSF studies show normal glucose,
b. Mucinous discharge lymphocytes 80% and organisms with a halo
c. Increase progesterone around it. The most likely organism is?
d. Ferning of cervical mucus a. EBV
e. Cyclic bleeding b. Staph Aureus
Ans: D c. Klebsiella
d. E. coli
25. Most common complication of IUCD is? e. Cryptococcus Neoformans
a. Squamous metaplasia in endocervix Ans: E
b. Purulent vaginal discharge
c. Degenerative changes in cervical 32. The artery palpated between trachea and
epithelium Infrahyoid is?
d. UTI a. Superior thyroid artery
e. Cervical polyp b. External carotid artery
Ans: B (Irregular vaginal bleed not in options) c. Common carotid
d. Internal carotid artery
26. A Proto oncogene is defined as? Ans: C
a. Abnormal genes causing cell
proliferation 33. Neurovascular bundle is present in?
b. Normal genes causing cell proliferation a. Between External and internal
c. Abnormal genes causing cell intercostal muscle
suppression b. Beneath lower border of upper rib
d. Normal genes causing cell suppression c. Between Innermost and endothoracic
Ans: B d. Beneath lower border of lower rib
e. Beneath upper border of lower rib
27. A 35-year male has anemia, loss of appetite, Ans: D
low grade fever died 3 months of illness.
Autopsy confirmed evidence of 34. The intercostal neurovascular bundle is
histoplasmosis. This most likely system located:
involved was? a. Above the upper border of the ribs
a. Respiratory b. Between the external and internal
b. CNS intercostals
c. GI c. Between the transverse thoracic and
d. Reticuloendothelial system subcostalis
e. Genitourinary d. Deep to the upper border of the ribs
Ans: D e. Just beneath the lower border of the ribs
28. Neck infection anterior to Pretracheal fascia Ans: E
will most likely spread to?
a. Posterior mediastinum 35. A lady heard her husband saying that the
b. Superior mediastinum neighbor loves her but do not confess which
c. Lateral mediastinum further proves his secret love. The lady tried
d. Posterior mediastinum contacting the neighbor but he refused which
e. Anterior mediastinum makes her sad. The husband was right about
Ans: E the love. She is pre-occupied with this
thought and gets confused sometimes. The
29. Most common congenital cardiac anomaly is? lady is suffering from?
a. VSD a. Delusion
b. ASD b. Depression
c. TOF c. Bipolar
d. TGA d. Schizophrenia
Ans: A e. Schizoaffective syndrome
Ans: B
30. Major supply to posterior 1/3rd of the
interventricular septum is from? 36. A 40-year-old man resident of Dir, returned
a. LAD home from UAE because of illness after
13 | P a g e
RADIANT NOTES FCPS PEARLS 11th Edition-2022

spending 3 years in Sharjah, where he c. Stroma of iris


worked as a taxi driver. He became unwell d. Stroma of cilia
about a month ago with weakness, fever, e. Ciliary epithelium
diarrhea and weight loss. He was investigated Ans: E
in Sharjah hospital. He was pale, febrile and
emaciated, with mild jaundice, generalized 43. Costal cartilage is composed of?
lymphadenopathy and hepatosplenomegaly. a. Chondral
The most likely diagnosis would be: b. Elastic
a. AIDS c. Fibrous
b. Leishmaniasis d. Hyaline
c. Malaria Ans: D
d. Tuberculosis
e. Yellow fever 44. A 38-week pregnant woman visited her family
Ans: A in a remote area after which she develops
jaundice suddenly. Which type of hepatitis is
37. A 20 years old girl with diabetes type 1 is most likely prevalent in the pregnant women?
brought to ER in a semi-comatose condition. a. Hep E
An injection of insulin given to her will raise b. Hep A
her? c. Hep B
a. Breathing rate d. Hep C
b. Urine glucose concertation e. Hep D
c. pH of blood Ans: A
d. Blood glucose concentration
e. Blood K+ concentration 
Most Common hepatitis, =HAV
Ans: C 
Remote area= HEV
38. Diabetes insipidus causes a fall in the: 
HDV has the highest mortality rate at 20%
a. Extracellular but not intracellular fluid 
Lethal in pregnancy =HEV
volume 
Spread through Oro-fecal= HAV>HEV
b. Extracellular fluid osmolality 
Hepatocellular carcinoma = HBV> HCV (in
c. Intracellular fluid osmolality Pakistan Hep C is the most common cause)
d. Osmolality of the urine  Hepatitis B-----Associated with Polyarteritis
e. Reabsorption of water from the proximal nodosa, Aplastic anemia
tubules  Hepatitis C ---- Associated with NHL, ITP,
Ans: D autoimmune hemolytic anemia
39. Fascicles are enclosed in a sheath called? 45. A man presented with right hemiplegia and
a. Endomysium deviation of mouth to left and drooling of
b. Epimysium water on swallowing. The lesion is in?
c. Endothelium a. Left internal capsule
d. Fascia
b. Pons
e. Perimysium
c. Midbrain
Ans: E
d. Medulla
40. After a gunshot injury to the cervical vertebra, e. Cerebellum
the patient now is experiencing respiratory Ans: A
distress. Which muscle is most likely
affected? 46. During the pharyngeal stage of swallowing:
a. Diaphragm a. Epiglottis remains stationary
b. Scalenus anterior
b. Larynx moves up and forward
c. Pectoralis major
c. Palatopharyngeal folds move apart
d. Sternocleidomastoid
d. Soft palate moves downwards
e. Internal intercostal
Ans: A e. Vocal cords are abducted
Ans: B
41. HLA DR4 is associated with?
a. Rheumatoid arthritis 47. Physiological dead space is increased in?
b. Ankylosing Spondylitis a. Hypotension
c. Psoriatic arthritis b. Atelectasis
d. Reactive arthritis c. Pulmonary hypertension
e. Gouty arthritis d. Shunt
Ans: A Ans: A
42. Which structure is derived from Increased Dead space Decreased Dead
Neuroectoderm? space
a. Ciliary ganglion
 Emphysema Atelectasis
b. Corneal epithelium

14 | P a g e
RADIANT NOTES FCPS PEARLS 11th Edition-2022

 Neck Extension Neck Flexion 54. A guard working full shift presented with
 Bronchoconstriction Bronchodilation history of unconsciousness, fits, high grade
fever (105) with rigor, chills and dark urine for
 ETT intubation Tracheostomy
the past 2 days. CSF exam unremarkable.
 Standing Sleep Diagnosis?
 Smoking Hyperventilation a. TBM
 Hypotension Supine Position b. Pyogenic meningitis
c. Polio
48. Thoracic duct drains into?
d. Cerebral Malaria
a. Left brachiocephalic vein
e. Leishmaniasis
b. Junction of left subclavian and internal
Ans: D
jugular vein
c. Right brachiocephalic vein
55. A postmenopausal woman is on HRT for
d. Left subclavian
postmenopausal related osteoporosis. What
e. Right internal jugular
complication she is at risk of?
Ans: B
a. Thromboembolism
49. A 40-year female brought to ER with severe b. Breast Ca
abdominal pain was diagnosed as acute c. Cervical Ca
biliary colic. Which drug, besides Morphine d. Obesity
will provide good analgesic relief? e. Anxiety
a. Pethidine Ans: A
b. Hyoscine butyl bromide
c. Diazepam 56. A patient died in ER with long standing
d. Paracetamol pulmonary congestion due to mitral stenosis.
e. Amoxicillin On autopsy, the alveoli will be lined by which
Ans: B of the following?
a. Type 1 pneumocytes
50. Which of the following is an important b. Types 2 pneumocytes
premalignant lesion leading to SCC? c. Lymphocytes
a. Actinic keratosis d. Heart failure macrophages
b. Dysplastic naves e. Eosinophils
c. Bowen disease Ans: D
d. Leukoplakia
e. Hamartoma 57. Nissl bodies are mainly present in?
Ans: C a. Axon hillock
b. SER
 Premalignant lesion which leads to SCC is=
c. RER
Bowen's disease
d. Microtubules
 Premalignant lesion which leads to SCC and
needs EXCISION is= Actinic keratosis e. Microfilament
Ans: C
51. One of the first structural changes evident in
reversible ischemic cell injury is:  Nissl bodies are absent in= Axon hillock
a. Atrophy  While Nissl bodies are mainly present
b. Cell blebs in=RER/Granular ER
c. Hypoplasia
58. The extracellular matrix and the cytoskeleton
d. Squamous metaplasia
communicate across the cell membrane
e. Swelling of Endoplasmic reticulum
through which of the following?
Ans: E
a. Proteoglycans
52. Specialty of Purkinje fibers in heart is? b. Cadherin
a. Contain numerous myofibrils c. Intermediate filaments
b. Pacemaker of the heart d. Microtubules
c. Specialized heart muscle cells e. Integrin
Ans: C Ans: E

53. A patient undergoes Radical mastectomy and 59. A 50-year-old male developed poor renal
now presents with painless swelling of arm. function 6 months after renal transplant. The
This is likely due to? renal function improved after immuno-
a. Nerve injury suppressive therapy. The most likely cause
b. Ischemia for poor function was:
a. Acute cellular rejection
c. Venous obstruction
b. Acute humoral rejection
d. Lymphatic obstruction
c. Chronic rejection
e. Inflammation
d. Cyclosporin toxicity
Ans: D
e. Hyperacute rejection

15 | P a g e
RADIANT NOTES FCPS PEARLS 11th Edition-2022

Ans: A e. MI
Ans: D
60. Treatment of anemia after total gastrectomy
is? 64. Baroreceptor reflex increases which of the
a. Oral iron supplements following?
b. Folic acid and oral Vit B12 a. TPR
c. IV Vit B12 b. MAP
d. Transfusion c. Brain ridge reflex
Ans: C d. Blood pressure
Ans: B
61. An 80-year female presented with skin
pigmentation, tingling sensation and unable 65. A patient had RTA after which he spoke
to maintain postural balance for the last 3 fluently but with little sense. The lesion is in?
months. Labs shows Hb 8, WBC 3.2x10^9, a. Broca
MCV 112 MCH 26, ferritin 600. What is the b. Motor cortex
likely diagnosis? c. Wernicke
a. Iron deficiency anemia d. Temporal lobe
b. Sideroblastic anemia e. Frontal lobe
c. Megaloblastic anemia Ans: C
d. Sickle cell anemia
e. Anemia of chronic disease 66. Lateral aspect of the foot is supplied by which
Ans: C of the following nerves?
a. L4
 Megaloblastic macrocytic anemia b. L5
(Hypersegmented neutrophils) c. S4
o B12 deficiency anemia d. S5
o Folic acid deficiency anemia e. S1
B12 def has neurological symptoms while folate Ans: E
don’t
 Non-megaloblastic macrocytic anemia (NO 67. All the short muscles of hand are supplied
hypersegmented neutrophils) by?
o Alcoholism a. Radial nerve
o Liver disease b. Median nerve
o Hypothyroidism c. Ulnar nerve
62. A known case of left ventricular d. T1
hypertrophy presented with significantly e. C5
increased end diastolic volume, increased Ans: D
pulmonary wedge pressure and increased
aortic pressure. There was early diastolic 68. A patient of chronic liver disease presented
murmur with BP 145/40. What is the likely with hematemesis. This blood is from
diagnosis? dilation of which vein?
a. AR a. Inferior epigastric vein
b. Tricuspid insufficiency b. Superior mesenteric vein
c. MS c. Right gastric vein
d. AS d. Portal vein
e. MR e. Left gastric vein
Ans: A Ans: E

 AR---wide pulse pressure, early diastolic  Portal ↔ Systemic sites


decrescendo murmur  Paraumbilical↔ small epigastric veins of
 AS--- narrow pulse pressure, Crescendo- the anterior abdominal wall.
decrescendo ejection systolic murmur  left gastric vein ↔azygous
 superior rectal vein↔ middle and inferior
 MS---- Mid-diastolic murmur---opening
rectal veins
snap
69. A 45-year male chronic smoker presented to
 MR-----blowing pansystolic murmur at
the ER with epigastric pain and hematemesis.
apex
He was diagnosed to have duodenal ulcer.
 MVP--- mid-systolic click The most likely cause of his bleeding would
 PDA---Machine like murmur be?
63. Non thrombotic bacterial endocarditis is a. Anterior duodenal ulcer with erosion of
frequently associated with? inferior pancreatic duodenal artery
a. SLE b. Anterior duodenal ulcer with erosion of
b. IHD pancreatic duodenal artery
c. Congenital heart disease c. Posterior duodenal ulcer with erosion of
d. Terminal neoplasm gastroduodenal artery

16 | P a g e
RADIANT NOTES FCPS PEARLS 11th Edition-2022

d. Posterior duodenal ulcer with erosion of making incision on Mc’burney’s point during
portal vein appendectomy is?
e. Posterior duodenal ulcer with erosion of a. Ilioinguinal nerve
right hepatic artery b. Genitofemoral nerve
Ans: C c. Deep circumflex femoral artery
d. Celiac nodes
Duodenal ulcers Gastric ulcers e. Iliohypogastric nerve
Duodenal ulcer pain Decreases Gastric ulcer Ans: E
with food pain Grows
with food 76. In a patient, damage to the nerve arising from
the dorsal brainstem will likely cause?
Hemorrhage  Hemorrhage 
a. Ptosis of the eye
(posterior>anterior).from from erosion of b. Loss of intorsion
erosion of Gastroduodenal left gastric c. Mydriasis
artery artery d. Cyclopegia
70. Pituitary tumor invades which bone? e. Loss of extrusion
a. Occipital Bone Ans: B
b. Parietal Bone
c. Temporal Bone 77. The conduction system of heart is located in?
d. Pterion a. Myocardium
e. Sphenoid Bone b. Endocardium
Ans: E c. Epicardium
d. Apex
71. A 20 years’ girl with Pheochromocytoma e. Sub endocardium
needs preoperative control of blood pressure. Ans: E
The most useful drug would be?
 SA node = Sub epicardium (upper portion of
a. Methyldopa Crista terminalis)
b. ACE inhibitors  AV node = Endocardium (located in
c. Beta blocker posteroinferior part of interatrial septum
d. Alpha 1 and beta blocker  Conducting system = Subendocardium
e. Diuretics
78. Stimulation of sympathetic nerve to the heart
Ans: D
produce?
72. What is the purpose of giving ATT drugs a. Shortening AV nodal delay
combined? b. Decreased coronary blood flow
a. To prevent the development of resistance c. Decrease stroke volume
b. They enhance the action constitute drugs d. Hyperpolarized SA fiber
c. To prevent side effects of individual drugs e. Decreased in conduction velocity through
d. For rapid action ventricle
e. To increase potency Ans: A
Ans: A
79. Autoregulation of kidney is done by?
73. A 36-year male of Lebanese ancestry is being a. Myogenic and Tubuloglomerular
treated for P. Vivax malaria. He experiences feedback system
severe fatigue, back pain and darkened urine. b. Bowmen’s hydrostatic and
Which one of the following antimalarial drug Tubuloglomerular feedback system
is most likely to have caused his symptoms? c. Tubulocapillary feedback
a. Pyrimethamine d. Renin angiotensin aldosterone system
b. Artemisinin e. Increase blood flow
c. Primaquine Ans: A
d. Chloroquine 80. Oral anticoagulant is monitored by?
e. Quinine a. APTT
Ans: C b. Platelet count
c. BT
74. Whole of the retina except rods and cones is d. CT
supplied by which vessel? e. PT
a. Short ciliary artery Ans: E (INR not in options)
b. Long posterior ciliary artery 81. Short term, transient action of angiotensin 2
c. Central retinal artery is?
d. Anterior choroidal artery a. Thirst stimulation
e. Short posterior ciliary artery b. Salt retention
Ans: C c. Aldosterone release
d. Diuresis
75. The structure most likely to be damaged while

17 | P a g e
RADIANT NOTES FCPS PEARLS 11th Edition-2022

e. Vasoconstriction decision for partial pancreatectomy was


Ans: E made. On histology which of the following
types of necrosis will be seen?
82. FRC is sum of? a. Fat necrosis
a. Vital capacity plus tidal volume b. Coagulative necrosis
b. Expiratory reserve volume plus residual c. Fibrinoid necrosis
volume d. Caseous necrosis
c. Vital capacity plus expiratory reserve e. Liquefactive necrosis
volume Ans: A
d. Residual volume plus respiratory
capacity  Brain = Liquefactive necrosis
e. Can be measured on spirometry  Solid organs (kidneys, heart and adrenal
Ans: B glands) = coagulative necrosis
 T.B = caseous necrosis
83. A baby born with a cystic swelling
 Pancreatitis= enzymatic fat necrosis
lumbosacral region with meningeal covering
 Breast trauma or abdominal trauma=
and NO neural tissue in it. The likely
diagnosis is? traumatic fat necrosis
a. Myelomeningocele  Polyarteritis nodosa (any vascultitis) =
b. Myeloschisis fibrinoid necrosis.
c. Meningocele 88. A man working in a cotton mill industry is
d. Spina bifida and myelomeningocele likely to suffer from?
e. Spina bifida and Meningocele a. Bagassosis
Ans: E b. Anthracosis
c. Farmers lung
84. A 60-year female has had increasing fatigue d. Byssinosis
for the past 6 months. On physical e. Silicosis
examination she has a palpable spleen tip. Ans: D
Laboratory studies show a WBC count of
189,000/microliter. The peripheral blood Farmer’s lung  Mouldy hay or Grain dust
smear shows many mature and immature Bird fancier’s lung  Avian excreta, proteins
myeloid cells present. Cytogenetic analysis and feathers
of cells obtained via bone marrow aspiration
will reveal a translocation of? Byssinosis  Textile industries
a. 14:18 89. Which PFTs findings are closely related to a
b. 12:21 patient of bronchial asthma?
c. 9:22 a. Residual volume = 1150
d. 11:22 b. Tidal volume = 350
e. 8:21 c. FEV1/FVC <65
Ans: C d. Vital capacity
85. A man is lying in a awake in quiet room. He e. No effect on FEV1
has closed eyes and is mentally relaxed. His Ans: C
electroencephalogram is likely to show which
90. Which pharmacological agent when given in
waves?
excess of higher dose causes Grade 4
a. Beta
hepatic encephalopathy?
b. Gamma
a. Amphetamine
c. Alpha
b. Streptomycin
d. Delta
c. Paracetamol
e. Theta
d. Flumazenil
Ans: C
e. Nitrates
86. When the hypothalamic temperature is set Ans: C
higher than the body temperature then which
of the following will decrease? 91. An elderly obese man with severe
a. Starvation atherosclerosis developed acute abdominal
b. Piloerection pain. on laparotomy small intestine was dark
c. Vasoconstriction purple in colour. Superior mesenteric artery
d. Sweating was atherosclerotic and thrombosed,
e. Shivering mesenteric vein patent. This small intestine
Ans: D pathology will be called?
a. Dry gangrene
87. A 45-year man presents with severe epigastric b. Red infarction
pain, nausea, vomiting, fever and increase in c. Volvulus formation
serum amylase and lipase with normal AST d. Wet gangrene
and ALT levels. A diagnosis of acute e. GIT bleeding
pancreatitis made. Due to unbearable pain, a
18 | P a g e
RADIANT NOTES FCPS PEARLS 11th Edition-2022

Ans: D a. Hydralazine
b. Labetalol
92. Which of the following increases pancreatic c. Amlodipine
secretion and releases Ca from sarcoplasm? d. Methyldopa
a. CCK e. Furosemide
b. Secretin Ans: D
c. Somatostatin
d. Nicotine 97. Which collagen type for tensile strength helps
e. VIP in wound healing?
Ans: A a. Type 2
b. Type 3
93. Sympathetic regulation occurs in which of the c. Type 1
following instead of local metabolites? d. Type 4
a. Liver Ans: C
b. Brain
c. Skin 98. A pregnant lady presents with jaundice and
d. Kidney deranged LFTs. The most likely cause is?
e. Spleen a. Hepatitis A
Ans: C b. Hepatitis B
Organ Factors Determining c. Hepatitis C
Autoregulation d. Hepatitis D
e. Hepatitis E
Heart  The most important local
Ans: A
(coronary metabolic factors are hypoxia
circulation) (↓O2) and adenosine 99. A young man presented with recurrent
Brain  The most important local episodes of syncope. These syncope
metabolic factors  CO2 (pH) episodes are related to reduce cerebral
Kidneys  Myogenic (muscle contracts perfusion when the arm ipsilateral to the
when stretched), and subclavian stenosis is exercised. Subclavian
tubuloglomerular feedback steal phenomenon (SSP) occurs when there
Skeletal  Local metabolites during is stenosis or occlusion of the subclavian
muscle exercise: CO2, H+, artery. In SSP which artery is stealing blood
(CHALK) Adenosine, Lactate, K+, and shunting this to arms?
 At rest: sympathetic tone a. External carotid
Skin  Sympathetic stimulation most b. Internal carotid
important mechanism for c. Thyrocervical trunk
temperature control d. Vertebral artery
94. A man had extensive burns completely e. Cerebral artery
damaging stratum corneum, spinosum and Ans: D
lucidum. The most likely acute complication 100. A CLD patient presenting with hematemesis.
will be? The preferred immediate treatment is?
a. Absent pressure sensation a. Inj Vit K
b. Acute Cutaneous water loss b. Formed Platelet Conc.
c. Hair follicle rupture c. Inj Transamine
d. Sebum d. Inj Octreotide
Ans: B e. Whole blood
Ans: D
95. A man involved in car accident suffered a
spinal injury resulting in quadriplegia with 101. The substance most likely to cause an
normal BP and respiration. 3 months later, he increase in arterial blood pressure would be:
came back with BP of 180/110 and mentioned a. Acetylcholine
that his BP lowers by stimulation of lower b. Alpha 1 agonist
part of abdomen and passing of urine. This c. Phenoxybenzamine
raise in BP is due to? d. Saralasin
a. Increased parasympathetic activity e. Spironolactone
b. Pain due to hypotonic bladder filling Ans: B
c. Activation of alpha receptors because of
release of catecholamine 102. An obese man feeling tightness in his collar
d. Increased reflex activity in recovery period shirt, falls on the side suddenly upon sharp
e. Cystitis movement of his head. Which of the following
Ans: C has occurred in this patient?
a. Dec venous compliance
96. A 30 year known hypertensive female came to b. Inc TPR
OPD with deranged indirect bilirubinemia. c. Increase venous compliance
Likely drug causing it is? d. Increase ANP
19 | P a g e
RADIANT NOTES FCPS PEARLS 11th Edition-2022

e. Increase contractility a. Stimulated by central receptor


Ans: A b. Arterial PO2 increases
c. Stimulated by carotid body
103. A Child who had episode of influenza d. Arterial oxygen content is kept in normal
pneumonia presented with cough thick range
yellow mucoid sputum along with cough. e. Arterial PO2 remains normal (not
Most likely causative organism is? decreased)
a. Klebsiella Ans: E
b. Streptococcus Pneumoniae
c. Pseudomonas Aeruginosa 110. A man after lifting heavy weight experiences
d. Staph Aureus sudden pain in his arm. O/E swelling in upper
Ans: D forearm is noted at the site where the muscle
is attached to radial tuberosity. The muscle
104. A student carries a study in a population likely injured is?
comparing symptoms in diseased individuals a. Biceps brachii
against lack of those symptoms in non- b. Brachioradialis
diseased. This type of study is called? c. Anconeus
a. Cross sectional d. Supinator
b. Interventional cohort e. Pronator
c. Cohort Ans: A
d. Case control
e. Randomized control 111. A 14-year-old boy gets his finger with a
Ans: D sharp nail. Which one of the following is
primary/early mediator of acute inflammatory
105. Tetanus toxoid toxin is a? response in this?
a. Endotoxin a. IL 1
b. Exotoxin b. Bradykinin
c. Neurotoxin
c. Prostaglandin
d. Erythrogenic toxin
Ans: C d. Platelet aggravating factor
e. Histamine
106. Posterior external arcuate fiber is the other Ans: E
name for?
112. A young patient brought by her mother to ER
a. Anterior spinocerebellar tract
with altered sensorium, abdominal pain,
b. Posterior spinocerebellar tract
excessive thirst, hyperventilation, frequent
c. Spinoreticular tract urination and fruity-scented breath. ABGs
d. Dorsal column show metabolic acidosis, urine ketones +ve.
e. Cuneo cerebellar tract What is the diagnosis?
Ans: E a. Salicylate poisoning
b. DKA
 Posterior external arcuate fibers [TA] that
c. Ethylene glycol poisoning
arise from cells in the accessory or lateral
d. Dehydration
cuneate nucleus and pass to the cerebellum
e. Renal tubular acidosis
107. The fastest conduction speed of cardiac Ans: B
impulse is observed in:
a. AV node 113. Erythropoietin is formed mainly by?
b. Atrial muscle a. Peritubular interstitial cells of kidney
c. Internodal pathways b. Perivenous hepatocytes
d. Purkinje system c. Juxtra glomerular cells
e. Ventricular muscle d. Bone marrow
Ans: D e. Spleen
Ans: A
108. Each sarcomere of human skeletal (striated)
muscle is a myofibrillar unit most likely 114. SA Node functions as the primary
between two: pacemaker of heart because:
a. H bands a. It is capable of generating impulse
b. Intercalated disks spontaneously
c. M bands b. Has rich sympathetic stimulation
d. Sets of transverse tubules c. Has poor cholinergic innervation
e. Z discs d. Its firing rate is faster than others
Ans: E e. Is located in atrium
Ans: D
109. In carbon monoxide poisoning, hyperoxia
does not occur due to?

20 | P a g e
RADIANT NOTES FCPS PEARLS 11th Edition-2022

115. A man during appendectomy developed high Ans: A


grade fever, tachycardia and muscle fatigue.
This occurs due to defect in? 122. Treatment of status epilepticus is?
a. Na/K pump a. Phenytoin
b. Ca++ sarcoplasm b. Lorazepam
c. Dihydropyridine receptor- c. Phenobarbitone
d. K/Cl channel d. Sodium valproate
e. Ryanodine receptor e. Lamotrigine
Ans: E Ans: B
116. Function of Prostaglandins E2, F2, D2 in
inflammation is? 123. A sprained ankle resulting from excessive
a. Vasoconstriction eversion most likely demonstrates which structure
b. Increase vascular permeability is damage?
c. Platelet aggregation a. Talofibular ligament
d. Decrease vascular permeability b. Tendon calcaneus
e. Vasodilation c. Deltoid ligament
Ans: E d. Interosseous ligament
Ans: C
117. Parietal lobe infarct will cause?
a. Prosopagnosia 124. Which of the following is a cause of edema
b. Athetosis in known IHD, HTN patient with heart failure?
c. Dysdidokinesia a. Increase hydrostatic pressure
d. Astereognosis b. Decrease oncotic pressure
e. Social behavior issues c. Increase hypertension
Ans: D d. Increase capillary permeability
e. Na retention
118. A lady had a difficult labor at home in village. Ans: A
She was brought to hospital with history of
p/v bleed & oozing from gums for the last 10 125. In drug overdose, the best route to
hours. Her CBC shows Hb: 6 g/dl, platelets administer antidote is?
30,000/cumm, TLC 24,000/cumm with a. IM
neutrophilia. The peripheral blood film shows b. IV
burr cells. Her PT & APTT were prolonged. c. Oral
Most likely cause is? d. Parenteral
a. ITP e. Sublingual
b. Postpartum hemorrhage Ans: B
c. Septicemia
d. Von will brand disease 124. An 8-month old infant present with
e. Disseminated intravascular coagulation alternating episodes of obstruction and
(DIC) passage of diarrheal stools. Radiological
Ans: E finding show dilated bowel segments.
119. Which hypophyseal structure receives Colonic biopsy finding reveal absence of
signals from the hypothalamus via the ganglion cell in the distended bowel segment.
hypophyseal portal vein? It is a case of?
a. Pars distalis a. Ischemic bowel disease
b. Adenohypophysis b. Meckel diverticulum
c. Neurohypophysis c. Hirschsprung’s disease
d. Pars intermedia d. Omphalocele
e. Supraoptic nucleus e. Intestinal Malrotation
Ans: B Ans: C
120. A tailor developed TB symptoms, XRAY B/L
hilar lymphadenopathy. The mode of 125. Drug that can be used for MRSA is?
transmission of his disease is? a. Aztreonam
b. Co amoxiclav
a. Inhaling aerosol droplets
c. Metronidazole
b. Food
d. Fusidic acid
c. Water
e. Co-Trimoxazole
Ans: A
Ans: D
121. DOC for absence seizure is?  In this MCQ there was no option of
a. Ethosuximide vancomycin or linezolid and among the
b. Sodium valproate options only Fusidic acid was the correct
c. Carbamazepine option
d. Diazepam

21 | P a g e
RADIANT NOTES FCPS PEARLS 11th Edition-2022

 MRSA remain sensitive to Fusidic acid (Ref: d. Isovolumetric relaxation


Wikipedia and NCBI) e. Slow filling of ventricles
126. Fine, voluntary skilled discrete movement of Ans: B
hands are controlled by?
a. Cortico-ponto cerebellar tract Heart Reason
b. Corticospinal tract Sound
c. Olivo-spinal tract S1 Isovolumic contraction, mitral and
d. Rubrospinal tract tricuspid valve closure
e. Spinothalamic tract S2 Isovolumic relaxation, aortic and
Ans: B pulmonary valve closure
S3 Also known as Gallop rhythm or
127. An old patient after abdominal surgery
develops abdominal distention. O/E he was ventricular gallop
pale and BP low and tachycardiac. The most due to rapid ventricular filling phase
likely diagnosis is? S4 In late diastole (“atrial kick”).
a. Hypovolemic shock Also called atrial gallop
b. Septic shock
133. Greenish discharge from a post-operative
c. Cardiogenic shock
wound is likely due to which organism?
d. Neurogenic shock
a. Staph Aureus
e. Ascites
b. Pseudomonas
Ans: A c. E. coli
128. Which of following is autosomal dominant d. Klebsiella
disorder? e. Proteus
a. Colon cancer Ans: B
b. Retinoblastoma Organisms Pigment Mnemonic
c. Hemophilia Actinomyces Yellow “sulfur” Israel has yellow
d. Alpha antitrypsin deficiency israelii granules sand
e. Hepatocellular Ca S. aureus Yellow Aureus (Latin) =
Ans: B (Both A and B are correct but B>A since pigment gold (yellow)
Hereditary non polyposis colorectal carcinoma P. blue-Green Aerugula is
(HNPCC) happens in 5% of cases rest are sporadic) aeruginosa pigment green.
134. The hormone that maintains water and
129. A patient had an axillary injury after which electrolytes balance is?
there was loss of flexion at elbow and a. Renin
supination. Nerve damaged is?
b. ADH
a. Musculocutaneous
c. Aldosterone
b. Radial
d. ANP
c. Median
e. Angiotensin 2
d. Ulnar Ans: C
e. Brachial
Ans: A 135. A 49-year man has a pulmonary embolism
130. Dopamine has inhibitory effects on? that completely blocks blood flow to his left
a. Estrogen lung. As a result, which of the following will
b. Prolactin occur?
c. GH a. Ventilation /perfusion ratio in the left lung
d. Insulin will be zero
e. TSH b. Systemic arterial po2 will be elevated
Ans: B c. V/Q ratio in the left lung will be lower
than right lung
 Dopamine Released from the hypothalamus
d. Alveolar po2 in the left lung will be
 Inhibits prolactin secretion that is why called
approximately equal to the po2 in the
prolactin-inhibiting factor (PIF).
inspired air
131. Nephrogenic DI causes? e. Alveolar po2 in the right lung will be
a. Increase ECF osmolarity approximately equal to the po2 in
b. Decrease ECF osmolarity venous blood
c. Urine osmolarity increase Ans: D
d. ECF volume increase
Ans: A 136. Which of the following is associated with
Down syndrome?
132. S3 heart sound is produced by?
a. UMNL
a. Isovolumetric contraction
b. Horse shoe kidney
b. Rapid filling of ventricle
c. Cystic hygroma
c. Atrial diastole
22 | P a g e
RADIANT NOTES FCPS PEARLS 11th Edition-2022

d. Alzheimer b. Potent antioxidant


Ans: D c. Endothelial protection
d. Maintain skin integrity
137. A patient was given drugs by a local quack e. Deficiency causes peripheral neuropathy
and is now jaundiced. Which enzyme raised Ans: B
due to liver parenchymal injury is?
a. AST 144. A 30 yr man with sore throat and cervical
b. GGT lymphadenopathy. CBC picture shows
c. LDH presence of atypical lymphocytes. The most
d. Alkaline phosphatase important test in this setting is?
e. ALT a. Monospot test
Ans: E b. Mantoux test
c. Lymph node biopsy
138. The proliferation of breast alveoli and d. Blood culture
lobules is mediated by which hormone? e. X-ray
a. Estrogen Ans: A
b. Prolactin
c. Progesterone 145. A child on list for an orthopedic surgical
d. Oxytocin procedure. The mother gives a strong
e. Growth hormone bleeding family hx in both male and females
Ans: C from maternal side. Labs shows PT normal,
APTT raised BT raised. Diagnosis?
139. A 65 yr presents with jaundice. On work up a. Factor IX deficiency
Hb 4.5, increase bilirubin, increase retic count b. Thalassemia
9%. Smear shows Polychromasia and c. Hemophilia
clumps. What test to confirm diagnosis? d. Von Willebrand disease
a. Coombs test e. Capillary damage
b. Hb electrophoresis Ans: D
c. Osmotic fragility test
d. G6PD 146. A student of final year MBBS witnessing a
e. Bone marrow biopsy surgery for the first time collapsed in OT due
Ans: A to?
a. Increase vascular permeability
140. An old male patient with DM and HTN have b. Decrease in total peripheral resistance
lower limb edema and bilateral effusion. The c. Decrease cardiac output
possible etiology is? d. Sympathetic shock
a. Hypoalbuminemia e. Increase heart rate
b. Sodium retention Ans: B
c. Lymphatic obstruction
d. Increase hydrostatic pressure 147. Myelinated nerve fibres has more speed of
Ans: A conduction than unmyelinated due to?
a. Saltatory movement
141. 50 years old presented in emergency b. Insulation
department with failing heart. Which drug of c. Continuous conduction
choice should be given? d. Increase myofibrils
a. Acetylcholine Ans: A
b. Adrenaline
c. Dopamine 148. The best advisable approach for a
d. GABA seropositive HIV infection is?
e. Atropine a. Repeat the test
Ans: C b. Declare AIDS
c. Test should be repeated by other
142. Which parameter decrease if arterial blood methods to confirm
pressure decreases? d. Report should be correlated with history
a. Heart rate e. Pt should be isolated
b. Neural activity of carotid sinus Ans: C
c. Venous return
d. Venous tone 149. A lady has severe colicky pain which was
e. Contractility relieved by giving Dicyclomine. This drug
Ans: B acts on?
a. Dopamine receptor
143. Which statement is true regarding Vitamin b. Nicotinic receptor
E? c. Muscarinic 3 receptor
a. Carbohydrate metabolism d. Adrenergic receptor

23 | P a g e
RADIANT NOTES FCPS PEARLS 11th Edition-2022

e. H1 antagonist Ans: E
Ans: C 156. A patient present with spontaneous
lacrimation during eating. This could be due
150. MOA of propylthiouracil? to misdirected regeneration of which of the
a. Absorption of calcium from stomach following nerves?
b. Inhibit release of iodine a. Auriculotemporal nerve
c. Inhibit synthesis of thyroid hormone b. Facial nerve
d. Inhibit action of T3 and T4 peripherally c. Lacrimal nerve
e. Inhibit secretion of Thyrotropin by
d. Trigeminal nerve
pituitary
Ans: C e. Vagus nerve
Ans: B
151. A pt. was brought to ER in state of massive
hemorrhage because of trauma and leaded 157. The left phrenic nerve descending between
hypotension baroreceptor compensation lead mediastinal pleura and the pericardium is
to? accompanied with?
a. Increase parasympathetic activity a. Internal thoracic artery
b. Increase arteriole tone b. Musculophrenic artery
c. Increased venous compliance c. Pericardiophrenic artery
d. Decrease venous compliance d. Right coronary artery
Ans: D e. Superior thoracic artery
Ans: C
152. A woman after hysterectomy for benign
ovarian cyst came presented with pneumonia 158. Anti-epileptics Benzodiazepines,
and was admitted. On sputum C/S gram barbiturates and many anticonvulsants exert
negative rods were identified. A 4 days of their action through which receptor?
admission she goes into shock. What is the a. Adrenergic
mechanism of this shock? b. Dopamine
a. Increase vascular permeability c. GABA
b. Renal tubular necrosis d. Glutamate
c. Electrolyte imbalance
e. Muscarinic
d. Increased vasoconstriction
e. Increases hydrostatic pressure Ans: C
Ans: A
159. A 30-year female with no CVS or
153. A patient underwent abdominal surgery, respiratory ailment presented with
after which he repeatedly required fluid complains of dyspnea on lying down.
resuscitation due to water and electrolyte O/E a swelling in front of her neck is
loss. He does not remember which part of the noted. Which anatomical structure likely
intestine was operated. The resection of to cause this dyspnea is?
which portion of gut has caused this a. Enlarged thyroid isthmus
condition? b. Retrosternal goiter
a. Jejunum c. Multinodular goiter
b. Rectum d. Colloid goiter
c. Ileum e. Solitary nodule goiter
d. Colon Ans: B
e. Duodenum
160. Which of the following is a premalignant lesion of
Ans: C
oral cavity?
154. Which of the following cells are first to reach a. Lichen planus
the site of acute inflammation? b. Paget disease
a. Monocytes c. Actinic keratosis
b. Neutrophils Ans: A
c. Lymphocytes
d. Eosinophils 161. The end-diastolic ventricular volume (EDV)
e. Histamine depends mainly on?
Ans: B a. Atrial contraction
b. Distensibility of ventricles
155. The most likely place in brain where major c. Duration of the diastole
blood vessels are found is? d. Venous return
a. Deep to pia matter
e. Ventricular contraction
b. Extradural space
Ans: D
c. Subdural space
d. Within pia matter 162. A person was blindfolded by the Doctor and
e. Subarachnoid space was asked to recognizes objects by their
24 | P a g e
RADIANT NOTES FCPS PEARLS 11th Edition-2022

shapes by hand. The pathway involved is? b. Macrophages


a. Secondary somesthetic area c. NK cells
b. Parieto-occipital temporal area d. B lymphocytes
c. Somatosensory 2 e. T lymphocytes
d. Somatosensory 1 Ans: C
e. Somatosensory association area 169. A 40-year male has a solid painless mass in
Ans: E his testis. His serum AFP and HCG are
normal. It was resected and patient treated
163. A woman suffers a severe head injury in an with radiotherapy. The most likely tumor is?
accident. Shortly afterwards, she becomes
a. Seminoma
polydipsic and polyuric. Her urine osmolarity
is 75 mOsm/L, and her serum osmolarity is b. Lymphoma
305 mOsm/L. Treatment with vasopressin c. Yolk sac tumor
causes an increase in her urine osmolarity to d. Germ cell tumor
450 mOsm/L. The most likely diagnosis is: Ans: A
a. Central diabetes insipidus
b. Nephrogenic diabetes insipidus 170. Which molecule is involved in cell-to-cell
c. Primary polydipsia adhesion, especially in embryological cell
d. Syndrome of inappropriate antidiuretic adhesion molecule?
hormone a. Selectin
e. Water deprivation b. Cadherin
Ans: A c. Integrin
d. ICAM
164. Gastric fundus secretes? e. VCAM
a. Mucous and Hcl Ans: B
b. Somatostatin and CCK
c. HCL and pepsin 171. A female gravida 3, para 2 with hx of
d. Intrinsic factor and Hcl previous 2 children with some karyotype
e. Hcl and somatostatin abnormality even though the female is
Ans: D physically normal presents to OPD with
concern of her pregnancy. The type of
165. The most likely change after resection of the abnormality is?
fundus and upper part of the corpus of the a. Deletion of q arm
stomach is: b. Isochromosome
a. Decreased gastrin production c. DiGeorge
b. Failure of the stomach to empty d. Robertsonian
Ans: D
c. Increase in BER (Basic Electrical
Rhythm)
172. A 50-year female pt undergoing
d. Increased secretion of pepsinogen chemotherapy due to CA breast presented
e. Loss of receptive relaxation of the with complains difficulty in swallowing due to
stomach a sore throat. O/E cheesy mucosal plaques in
Ans: E his mouth and white oropharynx
psuedohyphae are seen on pass stain of a
166. A patient with chronic obstructive lung mucosal smear. These findings are
disease has decreased: consistent with diagnosis of?
a. Airway resistance a. Cryptococcus
b. FEV1/FVC ratio b. Histoplasmosis
c. Forced vital capacity c. Rhinosporidiosis
d. Functional residual capacity d. Aspergillosis
e. Residual volume e. Candidiasis
Ans: B Ans: E

167. Which hormone increase activity of 173. In a post epileptic patient, the time taken
lipoprotein lipase? from fits to normal state is?
a. Growth hormone a. Post ictal
b. Insulin b. Aura
c. Thyroid hormone c. Clonic
d. Tonic
d. Glucagon
Ans: A
Ans: B
174. Macroglossia, hyperglycemia, HTN and
168. First line defense against Covid-19 virus and
polydipsia occur due to excess of which
tumor cells is provided by?
hormone?
a. Neutrophils
25 | P a g e
RADIANT NOTES FCPS PEARLS 11th Edition-2022

a. Corticotrophs d. Spleen and bone


b. Somatotrophs Ans: A
c. Prolactin
d. TRH 179. Biopsy of an unknown CA specimen was
e. Lactotrophs observed under microscope showing
Ans: B fibrovascular septa with scattered
calcifications giving a sand grain like
175. A patient complained of dryness of eyes. On appearance. The tissue being observed is?
examination there was decreased secretion a. Lung
of lacrimal glands. The most likely ganglia
b. Breast tissue
involved is:
c. Colon
a. Ciliary
b. Otic d. Thyroid
c. Pterygopalatine Ans: B
d. Submandibular
e. Superior cervical 180. A scientist experimenting on mice gave
Ans: C croton oil several times to the mice in a
week. How do promoters cause
176. A person is in complete shock state and his carcinogenesis?
compensatory mechanisms have failed. What a. DNA damage
type of acid base imbalance will be present in b. Inhibiting apoptosis
him? c. RNA damage
a. Metabolic acidosis with respiratory Ans: B
alkalosis
b. Metabolic Alkalosis with respiratory 181. Regarding hematopoiesis, correct is?
acidosis a. Long bones cause cell formation after
c. Metabolic acidosis with respiratory 20years
acidosis b. 75% myeloid stem cells
d. Metabolic acidosis c. Sternum stops hematopoiesis after 21
Ans: C years
d. 75% erythroid stem cells
 EXPLANATION: In shock there is Ans: C
Metabolic acidosis because urinary output
is almost zero, in order to compensated 182. One effect of androgen is to promote linear
that CO2 is exhaled from body so as to bone growth. This is transient because:
counteract the metabolic acidosis—means a. Androgens cause epiphyseal closure
respiratory alkalosis---but as per scenario b. Androgens increase the excretion rate of
compensation has failed so there will be calcium and phosphate ions
respiratory acidosis as well. c. Androgens slow the synthesis of
177. A Professor with a biopsy sample of lungs collagen
CA was asked by student about its growth d. Androgens stimulate bone resorption
e. Receptors for androgens are down
and size. The professor said it is 1 cm and it
regulated
needs to divide multiple times to make itself
Ans: A
about 10*9 cells. How many times should a
cell divide in order to reach 10*9 (1billion)? 183. A known patient of arthritis using drugs for 8
a. 50 times to 9 months presents with HB 11g/dl, MCV
b. 1000 times 101 fL, MCH = 30, bone marrow biopsy shows
c. 100000 times hypercellular. The likely cause is?
a. Aplastic anemia
d. 20 times
b. Hepatitis
e. 30 times c. Leukemia
Ans: E d. Lymphoma
e. Megaloblastic anemia
EXPLANATION: After 10 cell divisions 1000 copies
Ans: E
are produced from a single cell. Means 10 divisions
raise power by 10*3. So, 30 divisions will raise 184. Preganglionic sympathetic nervous system
power by 10*9. supplies?
a. Iris
178. B and T lymphocytes are necessary for
b. Sweet glands
normal immunity. The primary site of their
early maturation is? c. Kidney
a. Thymus and bone marrow d. Adrenal medulla
b. Spleen and lymph node Ans: D
c. Bone marrow and lymph node

26 | P a g e
RADIANT NOTES FCPS PEARLS 11th Edition-2022

185. Hallmark of inflammation is? a. Ameloblastoma


a. Vasodilation of vessels b. Osteomyelitis
b. Decrease capillary pressure c. Fibrous dysplasia
c. Movement of platelets in blood from d. Osteosarcoma
tissues Ans: C
Ans: A
192. Anterior hypothalamus maintains
186. A 24-years boy complains of jaundice, pallor, temperature effectively by?
and breathlessness and splenomegaly. He a. Cutaneous vasodilation
frequently visits the hospital OPD for b. Shivering
complains of repeated jaundice. The possible c. Vasoconstriction
cause could be? Ans: A
a. Sideroblastic anemia
b. Thalassemia 193. Basal bodies are derived from?
c. Hemolytic anemia a. Centrosome
d. Anemia of chronic disease b. Centrioles
Ans: C c. Pre-centrioles
d. Cilia
187. A blood sample from CRF patient taken for Ans: B
routine investigation was kept in a
refrigerator overnight. The sample was taken 194. Diabetes insipidus causes a fall in the?
to run tests next morning which showed a. Extracellular but not intracellular fluid
Sodium 140, K 6, LDH 600 and creatinine volume
raised. These results are most conclusive of?
b. Extracellular fluid osmolality
a. Progressive CA of kidney
c. Intracellular fluid osmolality
b. Chronic glomerulonephritis
d. Osmolality of the urine
c. End stage renal disease
e. Reabsorption of water from the proximal
d. Hemolytic reaction of sample tubules
Ans: D Ans: D
188. Biopsy of stomach shows presence of non- 195. When a nerve fiber placed in Na free solution
specialized gastric tissue with scatter goblet is stimulated, there is?
and columnar cells. This is an example of? a. Biphasic action potential
a. Dysplasia b. Compound action potential
b. Metaplasia c. Depolarization
c. Ca stomach d. Repolarization
d. Ulcer e. No action potential
e. Atrophy Ans: E
Ans: B
196. While analyzing biological data obtained
189. Primary oocytes remain arrested in the from a sample, Standard Deviation is
Diplotene which is a phase of? calculated along with the mean because it
a. Prophase indicates the:
b. Metaphase a. Quality of experimental design
c. Telophase b. Reliability of data
d. Anaphase c. Size of the sample
Ans: A d. Validity of conclusions
e. Variability amongst individual
190. A 45-year female patient develops difficulty observations
in language comprehension and intelligence. Ans: E
An MRI performed shows lesion most likely in
one of the following areas of brain: 197. The standard deviation shows which of the
a. Broca's area following entity?
b. Calcarine cortex a. Variability
c. Occipital cortex b. Prediction
d. Prefrontal motor cortex c. Central tendency
e. Wernicke's area d. Mode
Ans: E Ans: A

191. Patient with a painless maxillary swelling. On 198. Inactivated toxin is used as toxoid vaccine
radiology a radiolucent, radio-opaque lesion in:
with ground glass appearance is seen. The a. Measles
likely diagnosis is? b. Mumps

27 | P a g e
RADIANT NOTES FCPS PEARLS 11th Edition-2022

c. Rubella
d. Tetanus
e. Typhoid fever
Ans: D

199. Infant with dark color nappy (dark urine) and


homogentisate oxidase deficiency is likely to
suffer from?
a. Alkaptonuria
b. Phenylketonuria
c. Maple disease
d. Niemann-Pick disease
Ans: A

28 | P a g e
RADIANT NOTES FCPS PEARLS 11th Edition-2022

Medicine & Allied 30th Aug 2021, 6. A patient breathing in decompressed chamber
where the barometric pressure is 275 mmHg
Afternoon Shift (27000 altitude, 1/3 atmospheric). The oxygen
saturation will be?
a. 60%
1. Anti-epileptics Benzodiazepines, barbiturates b. 80%
and many anticonvulsants exert their action c. 20%
through which receptor? d. 100%
a. Adrenergic Ans: C
b. Dopamine
c. GABA  O2 content = P02/atm
d. Glutamate  Normal PO2 in atm=160mmhg
 >> Here 1/3rd so 160/3=53
e. Muscarinic
 >>Oxygen content of blood= 53/250 = 0.21
Ans: C  >> 0.21x100 =21
2. A pregnant hyperthyroid lady, during her 5th 7. A 35-year-old female with a butterfly malar rash
month of pregnancy, presents with and with positive ds-DNA presents with
thyrotoxicosis. Which anti-thyroid drug proteinuria and impaired renal function. The
should be given? most likely renal lesion is:
a. Propylthiouracil a. Amyloidosis
b. Radioiodine b. Lupus nephritis
c. Propranolol c. Pure interstitial nephritis
d. Prednisolone d. Renal thrombotic microangiopathy
e. Methimazole e. Tubo-interstitial disease due to NSAIDS
Ans: E Ans: B
3. A patient has a Blood Pressure of 70/40 mmHg 8. A female gravida 3, para 2 with hx of previous 2
& serum Lactate 30mg/100ml (Normal 6 to children with some karyotype abnormality
16). Cardiac output is 2L/min & CVP is 2cms even though the female is physically normal
of water. Most likely cause of this condition presents to OPD with concern of her
is? pregnancy. The type of abnormality is?
a. Cardiac Tamponade a. Deletion of q arm
b. CCF b. Isochromosome
c. Hypovolemic Shock c. DiGeorge
d. Pulmonary Embolism d. Robertsonian
e. Septic shock Ans: D
Ans: C
9. A newborn present with tetany. Investigation
 This question by CPSP is take from the book reveal serum Calcium 6.3 mg/dl. O/E a pan
Schwartz' Principles of Surgery: Self- systolic murmur is heard. A few days later,
Assessment and Board Review, Eighth ed: the newborn develops infections by
page 26, que 7) pneumocystis carinii, aspergillus fumigatus
4. Which molecule is involved in cell-to-cell and herpes simplex. He is likely suffering
adhesion, especially in embryological cell from?
adhesion molecule? a. 22q11 deletion syndrome
a. Selectin b. Adenosine deaminase deficiency
b. Cadherin c. B cell/plasma cell defect can produce
c. Integrin antibodies
d. ICAM d. Malaria
e. VCAM Ans: A
Ans: B
 DiGeorge syndrome--- Mnemonic: CATCH
5. A blood sample from CRF patient taken for 22
routine investigation was kept in a  Cardiac abnormalities, Abnormal facies, T-
refrigerator overnight. The sample was taken cell deficit because of Thymic hypoplasia,
to run tests next morning which showed Cleft palate, Hypocalcemia because of
Sodium 140, K 6, LDH 600 and creatinine
hypoparathyroidism, and microdeletion
raised. These results are most conclusive of?
22q11
a. Progressive CA of kidney
b. Chronic glomerulonephritis 10. Most common karyotype seen in Down's
c. End stage renal disease syndrome is:
d. Hemolytic reaction of sample a. Monosomy 21
Ans: D b. Trisomy 12
c. Trisomy 13

29 | P a g e
RADIANT NOTES FCPS PEARLS 11th Edition-2022

d. Trisomy 16 a. K ions
e. Trisomy 21 b. Vit D
Ans: E c. Vit A
d. Vit C
11. A child presents with mental retardation and e. Vit B12
low set ears, bilateral ptosis, epicanthal folds, Ans: E
and alternate esotropia. What is the
diagnosis? 17. A patient has end stage renal failure. He is
undergoing hemodialysis and has developed
a. Edward syndrome
normocytic normochromic anaemia. The
b. Down syndrome most appropriate therapy would be:
c. Achondroplasia a. Erythropoietin
d. Patua syndrome b. Ferrous sulphate
Ans: B
c. Folate
12. A 65-year-old patient presented with episodes d. Vitamin B12
of sharp shooting pain centered around the e. Vitamin B6
chin. He had been placed by his physician on Ans: A
tablet Tegretol 200 mg TDS, but of little relief.
On examination he was found to have 18. Macroglossia, hyperglycemia, HTN and
cutaneous eruptions in the distribution of the polydipsia occur due to excess of which
nerve. The primary cause of pain most likely hormone?
is: a. Corticotrophs
a. Atypical facial pain b. Somatotrophs
c. Prolactin
b. Atypical odontalgia
d. TRH
c. Herpes zoster e. Lactotrophs
d. Osteomyelitis Ans: B
e. Trigeminal neuralgia
Ans: C 19. A man is carried from sea level to 1200 height
in 2 days. After 12hrs of reaching the site, he
13. A boy complains of abdominal pain after developed dyspnea. The likely cause is?
trauma tenderness on palpitation, there is a. Secondary polycythemia
hematuria and proteinuria, Blood analysis b. Pulmonary edema
shows Iso blood cells. The trait is? c. Heart failure
a. Autosomal recessive d. Pneumothorax
b. Autosomal Dominant e. Pulmonary hypertension
c. X-linked Recessive Ans: B
d. Multifactorial
Ans: A  High altitude pulmonary edema (HAPE) is a
noncardiogenic pulmonary edema which
14. A patient has degenerative lesion in the typically occurs in lowlanders who ascend
caudate and putamen with deficiency of rapidly to altitudes greater than 2500-3000 m
GABA in the globus pallidus and substantia 20. A patient has episodes of fevers and chills
nigra. The condition disease, from which this which recur after 36-48 hours. Blood staining
patient suffers is: shows ring and crescent shape organism.
a. Athetosis The organism involved is?
b. Ballismus a. P. Malariae
c. Chorea b. P. Vivax
d. Hemiballismus c. P. Ovale
e. Parkinsonism d. P. Falciparum
Ans: C Ans: D

15. Mother had alcohol during pregnancy, baby is  Plasmodium falciparum is named for the
at risk of developing? crescent or falciform shape
a. Downs 21. During pregnancy, obliteration of uterine
b. Microcephaly cavity occurs due to fusion of?
c. Spina bifida a. Amnion and decidua capsularis
d. Klinefelter’s b. Chorion and amnion
e. Premature birth c. Decidua Parietalis and decidua
Ans: B capsularis
d. Decidua basalis and Capsularis
16. A 60-year had terminal ileum and ileocecal e. Chorion frondosum plus decidua basalis
junction resection. In the long run he is likely Ans: C
to have deficiency of?

30 | P a g e
RADIANT NOTES FCPS PEARLS 11th Edition-2022

22. A pt with paralysis of right side of body, 28. A male presented to OPD with 104 fever for
deviation of angle of mouth to the left and last 4 days and gives history of using Co-
drooling of saliva and the pt can close both trimoxazole. he has been passing cola-
eyes. Injury is at level of? colored urine for the last one day with
a. Supranuclear derange LFTs and increased unconjugated
b. Pontine nucleus bilirubin. What is the likely diagnosis?
c. Facial nucleus a. Drug induced jaundice
d. Petrous part of temporal bone b. Black water fever
e. Midbrain c. G6PD
Ans: A d. Paroxysmal nocturnal hemoglobinuria
23. Nucleus pulposus is derived from? e. Hemolytic uremic syndrome
a. Vertebrae Ans: C
b. Notochord
c. Vertebral canal 29. Mechanism of action of captopril?
d. Spleen a. Decrease angiotensin-2 activity
Ans: B b. Inhibit renin
c. Decrease cardiac output
24. Case with low HB, low TLC and low platelets. Ans: A
Which investigation is best?
30. Which structure is derived from the first
a. Retic count
pharyngeal arch is?
b. Bone marrow biopsy
a. Stapes
c. Serum Ferritin b. Stylohyoid ligament
d. Serum Iron c. Palatine
Ans: B d. Thymus
e. Muscle of mastication
25. Patient has lipoma om posterior aspect of Ans: E
Sternocleidomastoid muscle, 5cm relation to
sternoclavicular joint, during excision 31. Which of the following is a tumor suppressor
troublesome bleeding start due to? gene?
a. Internal jugular vein a. RAS
b. Common carotid artery b. BCL 2
c. External jugular vein c. C-myc
d. 2nd part of subclavian artery d. p53
e. Vertebral artery e. AFP
Ans: C Ans: D

32. Dementia, dermatitis and diarrhea occur due


26. A patient wished to take
to deficiency of which vitamin?
supplements/Vitamins for rejuvenation, while
surfing the internet she came across the a. Vitamin B3 (Niacin)
information that this vitamin deficiency is b. Vitamin B7 (Biotin)
similar to that of Vitamin B12 except anemia, c. Vitamin B6 (Thiamine)
what is the vitamin name? d. Vitamin C (Ascorbic acid)
a. Vitamin A Ans: A
b. Vitamin C
c. Vitamin D 33. A child suffering from high grade fever,
d. Vitamin E Mother took him to doctor, on examination
Ans: C there is petechial rash, on lab testing gram
+ive cocci found, what does it produce which
27. A 25-year female presents with lethargy and caused this?
irritability for the last 5 years. Labs revealed a. Pilus
Hb of 9.6, MCV 70 and MCHC low with normal b. Exotoxin
WBC and PLT count. She was prescribed Iron c. Endotoxin
supplements yet after 2 months she has Hb Ans: B
10. Which lab investigation should be ordered
to find the underlying cause of persistent 34. Ductus deferens ligated above scrotum, after
anemia? 2 days seminal fluid will contain?
a. Total iron binding capacity a. Semen only
b. HB electrophoresis b. Seminal fluid without sperm
c. Ferritin levels c. Prostatic and seminal fluid
d. Retic count d. Prostatic fluid
e. Bone marrow biopsy Ans: C
Ans: B
35. The process of unlocking the fully extended
31 | P a g e
RADIANT NOTES FCPS PEARLS 11th Edition-2022

knee in preparation for flexion requires initial 41. Vitamin for good memory?
contraction of which of the following? a. Vitamin B1
a. Gastrocnemius, soleus, and Plantaris b. Vitamin B3
muscles c. Vitamin B6
b. Hamstring muscles
d. Vitamin B12
c. Quadriceps femoris muscle
Ans: A
d. Sartorius muscle and short head of the
biceps femoris muscle
42. Inability to produce a urine pH less than 5.4
e. Popliteus muscle
even when challenged with ammonium is
Ans: E
found in:
 Locking= Quadriceps femoris a. Complete renal tubular acidosis
 Unlocking= Popliteus b. Distal renal tubular acidosis (type-1)
36. During a forced expiration, actively c. Glomerular insufficiency
contracting muscle is: d. Incomplete renal tubular acidosis
a. Abdominal muscles e. Proximal renal tubular acidosis (type-2)
b. Diaphragm Ans: B
c. External intercostals
d. Scalene 43. Left axis deviation on ECG is seen in?
e. Sternocleidomastoid a. RBBB
Ans: A b. Rt ventricular myocardial damage
c. Apex of heart on RT side
37. A student inserted a needle in thoracic cavity d. Pulmonary edema
for pleural tap at lower margin of rib. Which Ans: B
of the following will most likely be damage?
a. Artery 44. A man after lifting heavy weight experiences
b. Nerve sudden pain in his arm. O/E swelling in upper
c. Vein forearm is noted at the site where the muscle
d. Ganglion is attached to radial tuberosity. The muscle
likely injured is?
e. Muscle
Ans: B a. Biceps brachii
b. Brachioradialis
38. A young married female presented with c. Anconeus
history of recurrent abortions and deep vein d. Supinator
thrombosis. On investigation she had a e. Pronator
prolonged APTT which could not be Ans: A
corrected with normal plasma. The most
likely diagnosis is: 45. Which of the following is a naturally occurring
a. Antiphospholipid syndrome anti-thrombotic?
a. Dimercaprol
b. Antithrombin deficiency
b. Plasminogen
c. Factor V Leiden mutation
c. Heparin
d. Protein C deficiency d. Vit C
e. Protein S deficiency e. Warfarin
Ans: A Ans: C
39. One of the following tests is NOT for  Natural anticoagulant/Antithrombotic =
haemostatic function: Heparin
a. Activated partial thromboplastin time  While naturally occurring
b. Bleeding time fibrinolytic/thrombolytic= Plasminogen
c. Leukocyte count 46. The most common side effect of Levodopa?
d. Prothrombin time a. Glaucoma
e. Thrombin clotting time b. Mood changes
Ans: C c. Orthostatic hypotension
d. Acromegaly
40. The roots of Ansa cervicalis is derived from? e. Hypertension
a. Phrenic and vagal Ans: C
b. Hypoglossal nerve only
c. C2 and C3 47. A middle-aged woman presented with pain in
d. C1 epigastrium. She was diagnosed as a case of
gastric ulcer. This referred visceral pain is
e. Hypoglossal and C1, 2, 3
most likely mediated through:
Ans: E
a. 10th spinal nerve
b. 9th spinal nerve
32 | P a g e
RADIANT NOTES FCPS PEARLS 11th Edition-2022

c. Greater splanchnic nerve b. Ferritin


d. Least splanchnic nerve c. Haemoglobin electrophoresis
e. Lesser splanchnic nerve d. Iron and TIBC
Ans: C e. Reticulocyte count
Ans: C
48. A 48 years old female on routine cardiac
follow-up has BP 120/80 mmHg. ECG shows 53. Bone marrow biopsy is done from
prolonged PR interval and left axis deviation. a. Pubis
Echo did not demonstrate left ventricular b. Ischiopubic
hypertrophy. What is the diagnosis?
c. Ischial tuberosity
a. Bifascicular block
d. Iliac crest
b. Complete heart block Ans: D
c. Sick Sinus syndrome
d. Tri fascicular block 54. Pregnant lady with dead fetus, Oxytocin given
e. Uni fascicular block to female but she is not responding to it. This
Ans: A is likely due to?
a. Less receptors in myometrium
49. Regarding PR interval, which is correct? b. Less receptors in cervix
a. Decrease AV nodal conduction increase c. Decrease sensitivity
PR interval d. Decrease compliance
b. PR interval increases in WPW syndrome e. Decrease prostaglandin on myometrium
c. Due to atrial repolarization Ans: A
d. Represents interval b/w beginning of
55. Parasympathetic secretomotor nerve supply
atrial contraction and beginning of
to parotid gland:
ventricular contraction
Ans: A a. 5th Cranial nerve
b. 7th Cranial nerve
50. A patient taking Dicumarol can expect to have c. 9th Cranial nerve
which of the following symptoms? d. 11th Cranial nerve
a. Delayed blood clotting Ans: C
b. Enhanced blood clotting
c. Normal blood clotting 56. Aspirin overdose causes?
d. Plasmin deficiency a. Alkalosis
e. Platelet’s aggregation b. Coma
Ans: A c. Hypothermia
d. Deafness
50. Low voltage QRS complex is seen in? e. Gastric upset
a. Old MI Ans: A
b. Hypertension
c. IHD 57. A patient presents with homonymous
d. Pneumothorax hemianopia. The level of a lesion in visual
Ans: A pathway would be:
a. Lateral geniculate body
51. An unconscious boy of 11 year is brought to b. Optic chiasma
the emergency department of a hospital in a c. Optic nerve
state of shock. Doctor on duty is unable to d. Optic radiation
find any vein. The vein of lower limb most e. Optic tract
likely to be chosen for venesection is: Ans: E
a. Dorsal venous arch
b. Femoral vein 58. Lesion in the Lateral geniculate body will
c. Great saphenous vein cause?
d. Popliteal vein a. Rt homonymous hemianopia
e. Small saphenous vein b. Contralateral homonymous hemianopia
Ans: C c. Left homonymous hemianopia
d. Bitemporal hemianopia
52. A one-year-old child presented with e. Total blindness
progressive pallor with hepatosplenomegaly. Ans: B
Lab investigations showed Hb 5.0 mg/dl, MCV
51.5 fl, MCH 17.0 pg with RBCs showing 59. A person during surgery developed high
disturbed morphology. The most appropriate grade fever, tachycardia and muscle fatigue.
diagnostic investigation in this child would Which of the following is mainly involved in
be: this mechanism?
a. Bone marrow biopsy a. Ryanodine receptor
33 | P a g e
RADIANT NOTES FCPS PEARLS 11th Edition-2022

b. Na/K pump 64. Splitting of the second heart sound into two
c. Ca++ sarcoplasm components is enhanced by:
d. ACh gated ions a. Delayed closure of the aortic valve
Ans: A b. Delayed closure of the mitral valve
60. Which structure is responsible for sensing c. Early closure of the aortic valve
vibration? d. Early closure of the pulmonary valve
a. Ruffini e. Prolongation of atrial systole
b. Meissner Ans: C
c. Pacinian
d. Merkel 65. After surgical neck of humerus fracture, there
Ans: C is loss of abduction and proximal upper arm
lateral side sensation. Damage has occurred
Receptor Type Senses to?
Free Nerve  Pain a. Axillary nerve
Endings  Temperature b. Musculocutaneous nerve
Meissner  Fine/light touch c. Brachial artery
d. Ulnar nerve
Corpuscles  Two-point discrimination
e. Radial nerve
 Vibration (Low frequency) Ans: A
Pacinian  Pressure 66. The dose of Warfarin will have to be modified
Corpuscles  Vibration (High frequency) if the patient is receiving any of the following
Merkel Discs  Pressure drugs:
 Deep static touch (e.g., a. Amoxicillin
shapes, edges). b. Cimetidine
c. Ibuprofen
 Position sense (location)
d. Isoniazid
Ruffini  Pressure
e. Prednisolone
Corpuscles  Slippage of objects along Ans B
surface of skin,
 Joint angle change 67. Immediate reversal of Warfarin toxicity is by?
 Detects warm temperature a. Heparin
Krause bulb  Detects Cold b. Vit K
temperatures c. FFP
d. Cryoprecipitate
e. Protamine sulphate
Ans: C
61. The most common route of infection of H.B.V.
and H.C.V. is:  Remember that for warfarin toxicity both are
a. Blood transfusion used Vitamin K gives delayed effect, while
b. By direct contact with a patient fresh frozen plasma gives immediate effect.
c. Injection with contaminated needles
68. Scenario of a woman with invasive ductal
d. Through air pollution carcinoma. Histology shows glands, fibrous
e. Through water tissue and Desmoplasia. What does the term
Ans: C Desmoplasia means?
a. Proliferation of glandular tissue
62. Most common organism spread by blood
b. Metastatic invasion in surrounding
transfusion and its products?
structure
a. Hepatitis A virus c. Presence of abnormal tissue
b. Hepatitis B virus d. Proliferation of non-glandular fibrous
c. Hepatitis C virus connective tissue
d. Herpes zoster Ans: D
e. H pylori
Ans: B 69. The serology test for Aspergillus?
a. Galactosidase
63. Most common opportunistic infection after
b. Galactomannan
kidney transplant is?
c. Galactoprotein
a. HSV
b. EBV d. Phospholipid A
c. HIV e. IL-6
d. Polyoma Ans: B
e. HPV
Ans: D 70. A 40-year man known HTN presented with
sudden severe chest pain, massive

34 | P a g e
RADIANT NOTES FCPS PEARLS 11th Edition-2022

hemorrhage and died in ER. He was found to 77. A person developed urine infection. Urine
have aortic dissection. The autopsy would culture plate yielded a greenish growth
show? having fruity smell. This organism was
a. Atheroma oxidase positive and non-lactose fermenting.
b. Medial necrosis The most appropriate drug in this case would
c. Intimal tear be:
d. Saccular aneurysm a. Ampicillin
e. Fusiform aneurysm b. Ceftazidime
Ans: B c. Ceftriaxone
d. Ciprofloxacin
71. Which of following is side effect of Morphine? e. Gentamycin
a. Hypotension Ans: D
b. Rashes
c. Constipation 78. Hemi section with total spinal section is
d. Diarrhea differentiated by?
e. Increase gall bladder secretion a. Paraplegia in extension
Ans: C b. Degree of reflex activity
c. Spinal shock
72. Most common manifestation of autoimmune d. Permanent sensory loss
disease? Ans: C
a. Hematological changes
b. Fever 79. CVP is increased by?
c. Myalgia a. Decrease blood volume
d. Arthritis b. Increase venous compliance
Ans: A c. Increase total peripheral resistance
d. Decrease heart rate
73. Investigation for Hashimotos thyroiditis? e. Increase blood pressure
a. Anti-thyroglobulin and anti TPO Ans: D
b. Anti TSH antibody
c. TFTs 80. Which of the following causes increase in
d. Anti-thyroglobulin Renin level?
Ans: A a. Increase plasma osmolarity
b. Increase sodium delivery to macula
74. A young woman complains of dry mouth and densa
dry eyes. Which investigation will give clue to c. Decreased stretch in right atrial
her diagnosis? mechanoreceptors
a. ANA d. Exercise
b. Anti SSA Ans: C
c. Anti-ANA
d. RA factor 81. Lymphatic flow is decreased by :
e. ANCA a. Increase capillary oncotic pressure
Ans: B b. Increase capillary permeability
c. Increase interstitial oncotic pressure
75. Squamocolumnar epithelium present 3cm d. Capillary permeability
above of cardiac end of esophagus. Ans: A
Diagnosis?
a. Normal variant 82. A firefighter develops blisters on arm with
b. Achalasia little dermis damage, what degree if burn it
c. Hiatus hernia is?
d. Barret esophagus a. 1st degree
e. Cock screw b. 2nd degree
Ans: D c. 3rd degree
d. 4th degree
76. Fine, voluntary discrete movement of hands Ans: B
are controlled by?
a. Corticoponto cerebellar tract 83. A child was having fever, cough and upper
b. Corticospinal tract lobe consolidation, he developed respiratory
c. Olivo-spinal tract distress. ETT cannot be passed as he had
d. Rubrospinal tract Acute laryngotracheobronchitis (croup), What
is the most likely cause of this notorious
e. Spinothalamic tract
condition?
Ans: B
a. Respiratory syncytial virus
35 | P a g e
RADIANT NOTES FCPS PEARLS 11th Edition-2022

b. Moraxella catarrhalis 90. Which of the following is a QUALITATIVE


c. Haemophilus Influenza increase in gene expression?
d. Staph aureus a. Gene amplification
e. Klebsiella b. Promotor sequence insertion
Ans: C (if Parainfluenza virus in options prefer that) c. Enhancer sequence expression
d. Point mutations
84. Adenocarcinoma of esophagus develops in? Ans: A
a. Corrosive stricture
91. Vasculitis which is of different age in different
b. Smoking
vessel?
c. Reflux esophagitis
a. Wegner
d. Alcohol abuse b. Microscopic polyangiitis
e. Barrett esophagus c. Giant cell
Ans: E d. Takayasu
e. PAN
85. Myelinated nerve fiber has more speed of Ans: E
conduction than unmyelinated due to?
a. Saltatory movement 92. During a forced expiration, actively
b. Insulation contracting muscle is:
c. Continuous conduction
a. Abdominal muscles
d. Slow conduction
b. Diaphragm
Ans: A
c. External intercostals
86. In IgA nephropathy the histopathological d. Scalene
report will be consistent with: e. Sternocleidomastoid
a. Fusion of podocytes Ans: A
b. Increased mesangial matrix
c. Segmental glomerulo sclerosis 93. Considering the functions of autonomic
d. Segmental sclerosis in glomeruli nervous system, following is the
parasympathetic response:
e. Thickening of glomerular basement
membrane a. Constriction of blood vessels in skeletal
Ans: B muscles
b. Decrease in the secretion of epinephrine
87. Pituitary tumor invades which bone? & norepinephrine
a. Occipital Bone c. Decrease in the secretion of sweat
b. Parietal Bone glands
c. Temporal Bone d. Dilatation of blood vessels in skin and
d. Pterion viscera
e. Sphenoid Bone e. Increase in the secretion of salivary
Ans: E glands
Ans: E
88. A patient arrives in the E/R after having
suffered severe head trauma in a motorcycle 94. Involvement of which part of the brain is in
accident. Radiographic studies of the head control of direct autonomic reflexes?
reveal a basilar skull fracture in the region of a. Cerebral cortex
the foramen ovale. Which of the following b. Medulla
functional losses would most likely be related c. Hypothalamus
to this injury? d. Thalamus
a. Upper 1/3 of face affected e. Midbrain
b. Middle 1/3 of face affected Ans: C
c. Lower 1/3 of face affected
d. Loss of abduction of the eye 95. A man and woman have a history of recurrent
e. Loss of sensation over the forehead pregnancies, their clinical and genetic history
Ans: C is unremarkable but all their pregnancies end
up in still birth, how will you investigate?
89. A man had extensive burns completely a. Western Blot
damaging stratum corneum, spinosum and b. Southern blot
lucidum. The most likely acute complication c. Tandme mass spectrometry
will be? d. Next generation sequencing (NGS)
a. Absent pressure sensation e. Karyotyping
b. Cutaneous water loss Ans: E
c. Hair follicle rupture
d. Sebum 96. Baby born with a sac like defect in the
Ans: B lumbosacral region with neural tissue in it,
36 | P a g e
RADIANT NOTES FCPS PEARLS 11th Edition-2022

diagnosis?  Megaloblastic macrocytic anemia


a. Myeloschisis (Hypersegmented neutrophils)
b. Myelomeningocele o B12 deficiency anemia
c. Meningocele o Folic acid deficiency anemia
d. Spina bifida Occulta B12 def has neurological symptoms while folate
Ans: B don’t
 Non-megaloblastic macrocytic anemia (NO
97. The genetic inheritance of Hemophilia is? hypersegmented neutrophils)
a. X linked dominant o Alcoholism
o Liver disease
b. X linked recessive
o Hypothyroidism
c. Autosomal dominant
103. A female with bilateral ptosis that worsens at the
d. Autosomal recessive end of the day. Confirmatory Diagnostic
Ans: B investigation should be?
a. Tensilon test
98. An old man wants to stop aging which vitamin b. MRI brain
had anti-aging properties? c. CT brain
a. Vitamin B d. Anti-acetyl antibody
b. Vitamin C e. EMG
c. Vitamin D Ans: E
d. Vitamin E  Myasthenia gravis Investigations: (Ref
e. Vitamin K MTB step 2 CK)
Ans: D  Best initial test: acetylcholine receptor
antibodies (80%–90% sensitive) ---For
99. Damage to the medullary gracile and cuneate patients without those antibodies, get anti-
nuclei can result into: MUSK antibodies (muscle-specific kinase).
a. Amorphosynthesis  Edrophonium: short-acting inhibitor of
b. Analgesia acetylcholinesterase.
 Most accurate test: Electromyography
c. Anesthesia
shows decreased strength with repetitive
d. Anhydrosis
stimulation.
e. Astereognosis  CT/ MRI to exclude thymoma (CT with
Ans: E contrast is best than MRI)
104. Most common cause of death in rheumatic
100. One of the following muscles is NOT
fever is?
innervated by branches of the femoral nerve:
a. Mitral stenosis
a. Gracilis b. Myocarditis
b. Pectineus c. Endocarditis
c. Rectus femoris d. Pericarditis
d. Sartorius e. Heart failure
e. Vastus medialis Ans: B
Ans: A
105. The fibrous pericardium together with
101. Gracilis muscle is supplied by? parietal layer of serous pericardium is
a. Obturator nerve innervated by:
b. Femoral nerve a. Intercostal nerves
c. Pudendal nerve b. Left recurrent laryngeal nerve
d. Iliohypogastric nerve c. Phrenic nerve
e. Sural nerve d. Thoracic sympathetic nerves
Ans: A e. Vagus nerve
Ans: C
102. A female presented with fatigue, pallor and
tingling sensation in feet. CBC shows Hb 5.6,
106. A known HTN lady presented with severe
TLC 3500 and platelets 1,20000. Peripheral
headache, vomiting and raised ICP. On
film showed oval macrocytes along with
lumbar puncture, the CSF was blood stained.
hyper segmented neutrophils. This is likely
What is the most likely cause?
caused due to deficiency of?
a. Subdural hemorrhage
a. Folate
b. Extradural hemorrhage
b. Vit B12 deficiency
c. Injury during procedure
c. Iron
d. Subarachnoid hemorrhage
d. Calcium
Ans: D
Ans: B
107. Which drug should be avoided in CKD
patient?

37 | P a g e
RADIANT NOTES FCPS PEARLS 11th Edition-2022

a. Ciprofloxacin e. Cerebellum
b. Dopamine Ans: B
c. Rifampicin
d. Gentamycin 114. Systemic sclerosis or scleroderma is
Ans: D characterized by:
108. A 30-year male died suddenly after a sudden a. Cardiac failure
rise on blood pressure. On autopsy, the b. Diarrhea
ruptured vessel completely lacked the tunica c. Excessive fibrosis throughout the body
media at the point of the aneurysm. The most d. Keratitis
likely type of aneurysm is? e. Renal involvement
a. Dissecting aneurysm Ans: C
b. Luetic aneurysm
c. Berry aneurysm 115. A female patient presented with butterfly
d. Marfan aneurysm rash on her face and proteinuria. The most
e. Mycotic aneurysm sensitive initial screening test will be:
Ans: C
a. Anti-mitochondrial antibody
109. Neurogenic shock is due to: b. Anti-DNA antibodies
a. Decreased blood volume c. Anti-SM antibodies
b. Decreased cardiac output d. Anti-nuclear antibodies
c. Emotional disturbance Ans: D
d. Loss of vasomotor tone
116. An HIV positive patient has a higher risk of
e. Parasympathetic stimulation
developing infection after a surgical
Ans: D
procedure because of:
a. Decrease in the number of B cells
110. A mother delivered a child at home, and
brought her ‘5 days’ baby to hospital with b. Increased secretion of interleukin-1
neck stiffness, vomiting and fever. The cause c. Loss of CD4+ T cells
of meningitis is? d. Loss of CD8+ T cells
a. E. coli e. Polyclonal B cells activation
b. Streptococcus agalactiae (Group B Ans: C
Strep)
c. Neisseria Meningitis 117. A 30-year-old woman has experienced
d. Pneumococcus myalgias for the past 3 months. On physical
examination she has 5/5 motor strength in all
e. Staph Aures
extremities. She has dullness to percussion
Ans: B
at lung bases. A chest x-ray shows bilateral
pleural effusions. Laboratory studies show a
111. A female with non-caseating granuloma and
positive antinuclear antibody test at a titer of
Hilar lymphadenopathy (sarcoidosis). The
1:1024. Her serum urea nitrogen is 30 mg/dL.
type of cells involved are?
A renal biopsy is performed and microscopic
a. Basophils examination shows a granular pattern of
b. T lymphocytes immunofluorescence staining with antibody
c. Eosinophils to complement component C1q. This pattern
d. Plasma cells is most typically produced as a consequence
e. Neutrophils of which of the following immunologic
Ans: B mechanisms?
a. Antigen-antibody complexes
112. The precentral gyrus and corticospinal tract b. Antiglomerular basement membrane
are essential for: antibody
a. Auditory function c. IgE coating mast cells
b. Kinesthesia d. Macrophage release of lymphokines
c. Olfaction e. Release of prostaglandins
d. Vision Ans: A
e. Voluntary movement
Ans: E 118. A 28 year old lady with diffuse goitre
presents with history of weight gain and
113. Increased neural activity before a skilled intolerance to cold weather. Pulse rate was
voluntary movement is first seen in? 64/min regular. The investigation most likely
a. Spinal motor neurons confirming Hashimoto Thyroiditis would be:
b. Cortical association area a. Antimicrosomal and thyroglobulin
c. Precentral motor cortex antibodies
d. Basal ganglia b. Fine needle aspiration
c. Human specific TSH receptor antibodies
38 | P a g e
RADIANT NOTES FCPS PEARLS 11th Edition-2022

d. Normal T3 and T4, elevated TSH b. Ehler Danlos syndrome


e. TRH test c. Kartagener's syndrome
Ans: A d. Tetralogy of Fallot
e. Truncus arteriosus
119. Microscopic examination of the endocervical Ans: E
biopsy of a 30 years old female revealed the
endocervical tissue lined partially by 125. Choriostoma or heterotropic nest of cells are
stratified squamous epithelium. This is due to microscopically normal cells or tissue that
a process of: are present in abnormal locations. The most
a. Anaplasia suitable example is:
b. Dysplasia a. Ovarian tissue in rectal mucosa
c. Hyperplasia b. Pancreatic tissue found in the
d. Metaplasia submucosa of stomach
e. Neoplasia c. Pancreatic tissue in oral cavity
Ans: D d. Squamous cell in the urinary bladder
e. Thyroid tissue at the base of tongue
120. The most important objective for a screening Ans: B
test in cancer diagnosis is:
a. Screening is repeated if the tumour is 126. A 66-year-old man has had increasing
insidious in onset dyspnea for about a year. He is a smoker. He
b. The disease is detected at an early is retired from the construction business.
stage There are some rales auscultated in both
c. The test is 100% sensitive and specific lungs on physical examination. A chest
d. The test is acceptable to screening radiograph reveals bilateral diaphragmatic
population pleural plaques as well as diffuse interstitial
e. The treatment outcome is better for early lung disease. A sputum cytology shows no
tumour atypical cells. Pulmonary function studies
Ans: B reveal a low FVC and a normal FEV1/FVC
ratio. These findings are most likely to
121. The tumour usually associated with nerve suggest prior exposure to which of the
sheath invasion is: following environmental agents?
a. Adenocarcinoma a. Asbestos crystals
b. Adenoid cystic carcinoma b. Beryllium
c. Carcinoma expleomorphic adenoma c. Cotton fibers
d. Haemangiopericytoma d. Fumes with iron particles
e. Mucoepidermoid carcinoma e. Silica dust
Ans: B Ans: A

122. Para-neoplastic syndrome is exhibited by 127. A 40-year-old previously healthy woman, a


one of the following tumours: non-smoker has had episodes of fever, non-
a. Adenocarcinoma stomach productive cough and dyspnea over the past
3 months. Her symptoms disappeared after a
b. Mesothelioma
month's vacation, but reappeared when she
c. Renal cell carcinoma returned home to take care of her canaries.
d. Small cell carcinoma lung On physical examination there are no
e. Thymoma abnormal findings. A chest radiograph shows
Ans: D fine diffuse and nodular infiltrates in all lung
fields. Her disease is most likely to be
123. Some of the neoplasms appear to develop produced via which of the following
from viral oncogenesis. Which of the mechanisms?
following neoplasms is most likely to arise in a. Antigen-antibody complex formation
this manner? b. Infection with Mycobacterium Kansasii
a. Hepatic angiosarcoma c. Langerhans cell proliferation
b. Prostatic adenocarcinoma d. Mast cell degranulation
c. Retinoblastoma e. Progressive interstitial fibrosis
d. Small cell carcinoma Ans: A
e. T cell Leukemia
Ans: E 128. In Libman-Sacks endocarditis, the heart
valves characteristically effected is/are:
124. The congenital anomaly which results from a. Aortic alone
failure of separation of the aorta from the b. Aortic and pulmonary
pulmonary artery is:
c. Mitral and aortic
a. Ebstein's malformation
d. Mitral and tricuspid
39 | P a g e
RADIANT NOTES FCPS PEARLS 11th Edition-2022

e. Tricuspid and pulmonary a. Factor VIII


Ans: D b. Factor IX
c. Factor X
129. The most important sign in eyes, in case of d. Fibrinogen
arteriosclerosis of the vessel wall due to e. Tissue thromboplastin
hypertension would be: Ans: E
a. Changes at arteriovenous crossings (AV
nipping) 135. A 4 years old boy was seen with history of
b. Fleshing spot repeated episodes of painful swollen knee.
c. Exudative retinal detachment His laboratory tests showed PT 11 secs,
d. Retinal hemorrhage APTT 60 secs, factor VIII 85 %, factor IX 7 %.
e. Angioid streaks The boy suffers from:
Ans: A a. D.I.C
b. Christmas disease
130. The most appropriate statement with regard c. Fibrinogen deficiency
to non-bacterial thrombotic endocarditis d. Haemophilia A
(Marantic endocarditis) is: e. Von Willebrand's disease
a. Bulky, vegetative lesions occur on the Ans: B
mitral valve
b. It is seen in association with neoplastic 136. A 70-year-old woman has been bed ridden
disease for 5 weeks following a cerebrovascular
c. It is seen in systemic lupus accident (CVA). She has the sudden onset of
erythematosus dyspnea, but has no further symptoms until
d. Thickened endocardium of the tricuspid two days later when she experiences left
valvular cusps often occurs sided pleuritic chest pain. A day later she
e. Women are most frequently affected suffers another CVA and dies. At autopsy,
Ans: B she is found to have a wedge-shaped area of
hemorrhage based on the pleura of the left
131. A mother brings a child to you because of lower lobe. Which of the following pathologic
pallor. X-ray skull shows "hair-on-end", findings in her pulmonary arterial branches is
appearance. The most appropriate she most likely to have?
investigation for diagnosis would be: a. Aspergillosis
a. Bone marrow examination b. Atherosclerosis
b. B. Bone scan c. Fat embolism
c. Haemoglobin electrophoresis d. Thromboembolism
d. Peripheral blood smear e. Vasculitis
e. X-ray of skeleton Ans: D
Ans: C
137. Surfactant activity in the lung is most
132. Microcytic hypochromic anemia will typically directly related to the prevention of:
occur in: a. Alveolar proteinosis
a. Aplastic anemia b. Atelectasis
b. Folic acid deficiency c. Emphysema
c. Hereditary spherocytosis d. Interstitial fibrosis
d. Iron deficiency e. Pneumonia
e. Sickle cell anemia Ans: B
Ans: D
138. A 30-year-old man gives a history of a sore
133. The erythroid stem cells of bone marrow are throat with fever followed by 6 weeks of
active in cell cycle and capable of self- malaise. On physical examination he has
renewal for a considerable period of time mildly tender generalized lymphadenopathy.
under the influence of: A cervical lymph node biopsy is performed
and on microscopic examination shows a
a. Androgens
diffuse polyclonal hyperplasia. Which of the
b. Erythropoietin
following is the most likely diagnosis?
c. Growth factors
a. Brucellosis
d. Growth hormone
b. Hodgkin disease
e. Thyroxine
c. Human immunodeficiency virus infection
Ans: B
d. Infectious mononucleosis
134. The most likely thrombogenic agent which e. Lymphocytic lymphoma
initiates the process of disseminated Ans: D
intravascular coagulation (DIC) is:
139. A diagnosis of inclusion conjunctivitis is
40 | P a g e
RADIANT NOTES FCPS PEARLS 11th Edition-2022

made in a newborn to a mother with cervical


infection. The most likely organism
responsible is:
a. Adeno-virus
b. Chlamydia trachomatis
c. Fungus
d. Herpes simplex virus
e. Staphylococcus aureus
Ans: B

140. A 35-year-old man has noted several 1 to 2


cm reddish purple, nodular lesions present
on the skin of his right arm which have
increased in size and number over the past 3
months. The lesions do not itch and are not
painful. He has had a watery diarrhea for the
past month. On physical examination he has
generalized lymphadenopathy and oral
thrush. Which of the following infections is
most likely to be related to the appearance of
these skin lesions?
a. Candida albicans
b. Human herpesvirus 8
c. Mycobacterium tuberculosis
d. Pneumocystis carinii
e. Pseudomonas aeruginosa
Ans: B

141. The most likely feature of streptococcus


pneumoniae is:
a. It is a common aetiological agent in
meningitis
b. It is a strict anaerobe
c. It is always sensitive to penicillin
d. It produces Beta haemolysis on blood
agar
e. On Gram staining appears as Gram
positive cocci in clumps
Ans: A

41 | P a g e

You might also like